Get the Reddit app

DataIsBeautiful is for visualizations that effectively convey information. Aesthetics are an important part of information visualization, but pretty pictures are not the sole aim of this subreddit.

Average length of PhD dissertations by major

By continuing, you agree to our User Agreement and acknowledge that you understand the Privacy Policy .

Enter the 6-digit code from your authenticator app

You’ve set up two-factor authentication for this account.

Enter a 6-digit backup code

Create your username and password.

Reddit is anonymous, so your username is what you’ll go by here. Choose wisely—because once you get a name, you can’t change it.

Reset your password

Enter your email address or username and we’ll send you a link to reset your password

Check your inbox

An email with a link to reset your password was sent to the email address associated with your account

Choose a Reddit account to continue

biology phd thesis length

  • How Long Is a PhD Thesis?
  • Doing a PhD

It’s no secret that one of the most challenging aspects of a PhD degree is the volume of work that goes into writing your thesis . So this raises the question, exactly how long is a thesis?

Unfortunately, there’s no one size fits all answer to this question. However, from the analysis of over 100 PhD theses, the average thesis length is between 80,000 and 100,000 words. A further analysis of 1000 PhD thesis shows the average number of pages to be 204 . In reality, the actual word count for each PhD thesis will depend on the specific subject and the university it is being hosted by. This is because universities set their own word length requirements, with most found to be opting for around 100,000.

To find out more about how these word limits differ between universities, how the average word count from STEM thesis differ from non-STEM thesis and a more detailed breakdown from the analysis of over 1000 PhDs, carry on reading the below.

Word Count Differences Between Universities

For any PhD student writing a thesis, they will find that their document will be subject to a word limit set by their university. In nearly all cases, the limit only concerns the maximum number of words and doesn’t place any restrictions on the minimum word limit. The reason for this is that the student will be expected to write their thesis with the aim of clearly explaining their research, and so it is up to the student to determine what he deems appropriate.

Saying this, it is well accepted amongst PhD students and supervisors that the absence of a lower limit doesn’t suggest that a thesis can be ‘light’. Your thesis will focus on several years worth of original research and explore new ideas, theories or concepts. Besides this, your thesis will need to cover a wide range of topics such as your literature review, research methodology, results and conclusion. Therefore, your examiners will expect the length of your thesis to be proportional to convey all this information to a sufficient level.

Selecting a handful of universities at random, they state the following thesis word limits on their website:

  • University of Edinburgh: 100,000
  • University of Exeter: 100,000
  • University of Leister: 80,000
  • University of Bath: 80,000
  • University of Warwick: 70,000

The above universities set upper word limits that apply across the board, however, some universities, such as the University of Birmingham and the University of Sheffield, set different word limits for different departments. For example, the University of Sheffield adopts these limits:

  • Arts & Humanities: 75,000
  • Medicine, Dentistry & Health: 75,000
  • Science: 80,000
  • Social Sciences: 75,000-100,000

Although there’s a range of limit, it’s safe to say that the majority fall within the 80,000 to 100,000 bracket.

Word Count Based on Data from past Theses

A poll of 149 postdocs.

In mid-2019, Dr Eva Lantsoght, a published author, academic blogger and Structural Engineering Professor, conducted a poll which asked postgraduate doctoral students to share the length of their final thesis. 149 PostDoc students responded to the survey, with the majority reporting a length falling within the ‘80,000 – 120,000 words’ bracket as seen below.

DiscoverPhDs_How-long-is-a-PhD-Thesis_Poll

Analysis of 1000 PhD Theses

Over a three-year time period, Dr Ian Brailsford, a then Postgraduate Learning Adviser at the University of Auckland, analysed 1000 doctoral thesis submitted to his university’s library. The PhD theses which formed the basis of his analysis were produced between 2008 to 2017 and showed:

  • Average number of pages = 204
  • Median number of pages = 198
  • Average number of chapters = 7.6

We should note that the above metrics only cover the content falling within the main body of the thesis. This includes the introduction, literature review, methods section, results chapter, discussions and conclusions. All other sections, such as the title page, abstract, table of contents, acknowledgements, bibliography and appendices were omitted from the count.

Although it’s impossible to draw the exact word count from the number of pages alone, by using the universities recommended format of 12pt Times New Roman and 1.5 lines spacing, and assuming 10% of the main body are figures and footnotes, this equates to an average main body of 52,000 words.

STEM vs Non-STEM

As part of Dr Ian Brailsford’s analysis, he also compared the length of STEM doctorate theses to non-STEM theses. He found that STEM theses tended to be shorter. In fact, he found STEM theses to have a medium page length of 159 whilst non-STEM theses had a medium of around 223 pages. This is a 40% increase in average length!

Can You Exceed the Word Count?

Whilst most universities will allow you to go over the word count if you need to, it comes with the caveat that you must have a very strong reason for needing to do so. Besides this, your supervisor will also need to support your request. This is to acknowledge that they have reviewed your situation and agree that exceeding the word limit will be absolutely necessary to avoid detriment unnecessary detriment to your work.

This means that whilst it is possible to submit a thesis over 100,000 words or more, it’s unlikely that your research project will need to.

How Does This Compare to a Masters Dissertation?

The average Masters dissertation length is approximately 20,000 words whilst a thesis is 4 to 5 times this length at approximately 80,000 – 100,000.

The key reason for this difference is because of the level of knowledge they convey. A Master’s dissertation focuses on concluding from existing knowledge whilst a PhD thesis focuses on drawing a conclusion from new knowledge. As a result, the thesis is significantly longer as the new knowledge needs to be well documented so it can be verified, disseminated and used to shape future research.

Finding a PhD has never been this easy – search for a PhD by keyword, location or academic area of interest.

Related Reading

Unfortunately, the completion of your thesis doesn’t mark the end of your degree just yet. Once you submit your thesis, it’s time to start preparing for your viva – the all-to-fun thesis defence interview! To help you prepare for this, we’ve produced a helpful guide which you can read here: The Complete Guide to PhD Vivas.

Browse PhDs Now

Join thousands of students.

Join thousands of other students and stay up to date with the latest PhD programmes, funding opportunities and advice.

Doctor of Philosophy (PhD) in Biology

The PhD in Biology is a research degree requiring graduate-level coursework, completion of a dissertation, and two semesters of participation in teaching (usually as a teaching fellow in laboratory or discussion sections of lecture courses led by Biology faculty). For most students, obtaining this degree typically involves five or more years of full-time study.

A summary of Biology PhD student expectations by year can be found here . Full details can be found in the Graduate Program Guide .

The Biology Department guarantees support for five years for all PhD students, contingent on satisfactory performance in the program.

How to Apply    Frequently Asked Questions

Learning Outcomes

1. Demonstrate academic mastery in one of three areas of Biology: Ecology, Behavior & Evolution; Neurobiology; or Cellular & Molecular Biology.

2. Attain research expertise , including grant writing experience, and complete original research that advances a specific field of study within one of three broad subject areas represented in the department: Ecology, Behavior & Evolution; Neurobiology; or Cellular & Molecular Biology.

3. Attain teaching experience and expertise in one of three broad areas of Biology: Ecology, Behavior & Evolution; Neurobiology; or Cellular & Molecular Biology.

4. Attain the skills and qualifications needed for employment in an academic, government, or private sector position related to the life sciences.

Coursework Requirements

Students must complete 64 credits with a minimum grade point average of 3.0; at least 32 of these credits must be accrued from lecture, laboratory, or seminar courses. Students with prior graduate work may be able to transfer course credits.  See the  Graduate School of Arts & Sciences (GRS) Transfer of Credits policy  for more details. 

  • 2 semesters of Progress in Research Seminars (2 credits each):

Cell & Molecular Biology:  BI 583 & BI 584 (CM section)

Ecology, Behavior & Evolution: BI 579 & BI 580

Neurobiology:   BI 533 & BI 5834

  • 1 semester grant-writing course (2 credits):

Cell & Molecular Biology:  BI 581

Ecology, Behavior & Evolution: BI 671

Neurobiology: BI 581

  • 1 semester pedagogy course (1 credit): All Biology PhD students take  BI 697
  • 1 quantitative course (3–4 credits): from a list of recommended courses .
  • Additional courses vary by specific track below:

Cell & Molecular Biology

1. GRS BI 791/GRS BI 792 Graduate Rotation Credits (2 credits each/4 credits total)

2. GRS BI 753 Advanced Molecular Biology  (4 credits)

3. GRS MB 721 Graduate Biochemistry  (4 credits)

4. GRS BI 735 Advanced Cell Biology  (4 credits)

5. Two electives (8 credits, 500-level and above), see the Graduate Program Guide for recommendations

6. Research credits, CAS BI 925/CAS BI 926  (remaining credits)

Ecology, Behavior & Evolution

1. Six electives  (24 credits, 500-level and above), see the Graduate Program Guide for recommendations

2. Research credits, CAS BI 911/CAS BI 912  (remaining credits)

Ecology, Behavior & Evolution PhD candidate coursework is highly variable. Students, in consultation with advisors, develop a plan of coursework and research. Students are required to take a minimum of 32 credits of coursework. The remainder of the credits should be research.

Neurobiology

2. GRS BI 755 Cellular and Systems Neuroscience  (4 credits)

3. GRS BI 741 Neural Systems: Functional Circuit Analysis  (4 credits)

4. Four electives  (16 credits, 500-level and above), see the Graduate Program Guide for recommendations

5. Research credits, CAS BI 939/CAS BI 940  (remaining credits)

Teaching Requirement

The department requires a minimum of two semesters of teaching as part of the Doctor of Philosophy program. During the first semester of teaching, students are required to enroll in our first-year seminar course, GRS BI 697 A Bridge to Knowledge . The course provides guidance and training on pedagogy and other aspects of graduate school.

Qualifying Examination

The qualifying examination must be completed no later than six semesters after matriculation. In most graduate curricula in the department, this consists of a research proposal—often in the form of a grant application—which the student submits to their committee and subsequently defends in an oral presentation. In the  Cell & Molecular Biology and Ecology, Behavior & Evolution curricula , this is preceded by a comprehensive written examination testing the student’s general background from coursework.

Dissertation & Final Oral Examination

Candidates shall demonstrate their abilities for independent study in a dissertation representing original research or creative scholarship. A prospectus for the dissertation must be completed and approved by the readers, the Director of Graduate Studies, and the Biology Department Chair. Candidates must undergo a final oral examination in which they defend their dissertation as a valuable contribution to knowledge in their field and demonstrate a mastery of their field of specialization in relation to their dissertation. All portions of the dissertation and final oral examination must be completed as outlined in the  GRS General Requirements for the Doctor of Philosophy Degree . The results of the dissertation must be presented at a department colloquium.

Graduation Timeline

Forms and additional information about PhD graduation can be found on the GRS website .

9 – 12 months before proposed graduation date

  • Dissertation Prospectus & approval form due to Graduate Program Specialist for departmental review and submission to GRS

Semester prior to your intended graduation cycle

  • Intent to Graduate Form completed online

About 2 months before dissertation defense

  • Meet with Graduate Program Specialist, Director of Graduate Studies, and Faculty Advisor to review requirements for defense
  • Arrange for Special Service Appointments if committee members are not BU faculty
  • Send first draft of dissertation to readers

Once defense date is confirmed with committee

  • Reserve room(s) for public seminar and defense

At least three weeks prior to dissertation defense

  • Schedule of Final Oral Exam with Abstract Approval due to GRS
  • Properly formatted draft of dissertation submitted as PDF to [email protected]

At least two weeks prior to dissertation defense

  • Send dissertation to all committee members

At least one week prior to dissertation defense

  • Send program information to Graduate Program Specialist

See the Graduate Program Guide for final dates to submit dissertation to ETD

  • Submit final dissertation to ETD (online submission)
  • Complete electronic signature page and forward confirmation to Graduate Program Specialist along with a copy of your submitted dissertation

MS Degree (En Route to PhD)

Option one: A PhD student who has advanced to candidacy (as demonstrated by passing the PhD qualifying exam), and has completed 32 credits of graduate-level coursework (not including research) may apply to the Graduate School for an MS degree in Biology. This must be approved by the Director of Graduate Studies within the Biology Department. The student’s major professor will receive notification of this application process.

Option two: A PhD student who has taken, but has not advanced to candidacy based on the PhD qualifying examination, may still receive an MS degree. This student may receive a Coursework MS degree provided they have completed 32 credits of coursework (not including research credits). Alternatively, this student may receive a Scholarly Paper or Research Thesis MS degree if the written portion of the qualifying examination is adapted to ensure it is of sufficiently high quality for a MS degree, and approved by a majority of the qualifying exam committee and the Director of Graduate Studies.

Interdisciplinary Study Options

Biology PhD students have the option to participate in the Boston University Graduate Program in Urban Biogeoscience and Environmental Health (BU URBAN), the National Science Foundation Research Traineeship Program Understanding the Brain: Neurophotonics (NSF NRT UtB: Neurophotonics), and the Biogeoscience Advanced Graduate Certificate Program . These programs require separate applications in addition to the standard Biology PhD application; those interested in BU URBAN are encouraged pre-apply .

Time Limits

Officially, the PhD must be completed within seven years after the first registration for doctoral study. PhD degrees are conferred in either May, August, or January, as specified on the GRS website . In addition, the PhD candidacy expires after the fifth anniversary of passing the Qualifying Examination. Petitions to extend this deadline are possible at the discretion of the Director of Graduate Studies, the Department Chair, and the Dean of the Graduate School, and can be obtained from the Office of the Graduate School of Arts & Sciences.

The Biology Department guarantees support for five years for all PhD students, contingent on satisfactory performance in the program. PhD students are encouraged to apply for fellowships and grants at funding agencies. All domestic students should apply for NSF Graduate Research Fellowships in the Fall semester of their first or second year. 

Travel Grants may be available to assist students in their travel to professional scientific meetings; students presenting papers or posters on their research will receive first consideration.

Common Types of Funding:

Dean’s Fellowships: These are non-service fellowships allocated to first-year PhD students that do not have immediate teaching requirements.

Teaching Fellowships: These provide a stipend plus full tuition and fees for up to four full courses per semester plus a 2-credit teaching course. Teaching responsibilities usually require approximately 20 hours per week. Full or partial awards may be given.

Doctoral Research Fellowships: These awards are given to students who assist individual faculty with specific areas of research. These Research Fellowships provide a stipend and full tuition. The supervising faculty member determines the specific duties of the Research Fellow.

In addition to the above funding sources, several competitive Department awards and fellowships are available to graduate students in the Department of Biology.

Back to Top

University of Cambridge

Study at Cambridge

About the university, research at cambridge.

  • Undergraduate courses
  • Events and open days
  • Fees and finance
  • Postgraduate courses
  • How to apply
  • Postgraduate events
  • Fees and funding
  • International students
  • Continuing education
  • Executive and professional education
  • Courses in education
  • How the University and Colleges work
  • Term dates and calendars
  • Visiting the University
  • Annual reports
  • Equality and diversity
  • A global university
  • Public engagement
  • Give to Cambridge
  • For Cambridge students
  • For our researchers
  • Business and enterprise
  • Colleges & departments
  • Email & phone search
  • Museums & collections
  • Undergraduate and Postgraduate Taught
  • Postgraduate examinations
  • Writing, submitting and examination
  • PhD, EdD, MSc, MLitt
  • Cambridge students
  • New students overview
  • Pre-arrival courses
  • Student registration overview
  • Information for New Students overview
  • Step-by-step guide for new students
  • Information for Continuing Students overview
  • Step-by-step guide for continuing students
  • Frequently Asked Questions overview
  • Who needs to register
  • When to register
  • Received registration in error/not received registration email
  • Problems creating an account
  • Problems logging in
  • Problems with screen display
  • Personal details changed/incorrectly displayed
  • Course details changed/incorrectly displayed
  • Accessing email and other services
  • Miscellaneous questions
  • Contact Form
  • First few weeks
  • Manage your student information overview
  • Student record overview
  • Camsis overview
  • Extended Self-Service (ESS)
  • Logging into CamSIS
  • What CamSIS can do for you
  • Personal information overview
  • Changing your name
  • Changing Colleges
  • Residing outside the University's precincts
  • Applying for person(s) to join you in Cambridge
  • Postgraduate students overview
  • Code of Practice for Master's students
  • Code of Practice for Research Students
  • Postgraduate student information
  • Requirements for research degrees
  • Terms of study
  • Your progress
  • Rules and legal compliance overview
  • Freedom of speech
  • Public gatherings
  • Disclosure and barring service overview
  • Cambridge life overview
  • Student unions
  • Extra-curricular activities overview
  • Registering societies
  • Military, air, and sea training
  • Food and accommodation
  • Transport overview
  • Bicycles and boats
  • Your course overview
  • Undergraduate study
  • Postgraduate study overview
  • Changes to your student status (postgraduates only) overview
  • Applying for a change in your student status (postgraduates only)
  • Changing your mode of study
  • Withdrawing from the University
  • Allowance/exemption of research terms
  • Withdrawal from Study
  • Reinstatement
  • Changing your course registration
  • Changing your department/faculty
  • Changing your supervisor
  • Exemption from the University composition fee
  • Confirmation of Study: Academic Verification Letters
  • Extending your submission date
  • Medical intermission (postgraduates)
  • Non-medical intermission (postgraduates)
  • Returning from medical intermission
  • Working away
  • Working while you study
  • Postgraduate by Research Exam Information
  • Research passports
  • Engagement and feedback
  • Student elections
  • Graduation and what next? overview
  • Degree Ceremonies overview
  • The ceremony
  • Academical dress
  • Photography
  • Degree ceremony dates
  • Eligibility
  • The Cambridge MA
  • Degree certificates and transcripts overview
  • Academic Transcripts
  • Degree Certificates
  • After Graduation
  • Verification of Cambridge degrees
  • After your examination
  • Exams overview
  • Undergraduate and Postgraduate Taught overview
  • All students timetable
  • Undergraduate exam information overview
  • Postgraduate examinations overview
  • Examination access arrangements overview
  • Research programmes
  • Taught programmes
  • Writing, submitting and examination overview
  • PhD, EdD, MSc, MLitt overview
  • Research Best Practice
  • Preparing to submit your thesis
  • Submitting your thesis
  • Word limits
  • The oral examination (viva)
  • After the viva (oral examination)
  • After the examination overview
  • Degree approval and conferment overview
  • Final thesis submission
  • Examination allowances for certain Postgraduate degrees (except PhD, MSc, MLitt and MPhil by thesis degrees)
  • Requesting a review of the results of an examination (postgraduate qualifications)
  • Higher degrees overview
  • Higher doctorates
  • Bachelor of divinity
  • PhD under Special Regulations
  • Faith-provision in University exams
  • Publication of Results
  • Exam Support
  • Postgraduate by Research
  • EAMC overview
  • Annual Reports of the EAMC
  • Dates of meetings
  • Frequently asked questions
  • Guidance notes and application forms
  • Resources overview
  • Build your skills overview
  • Skills Homepage
  • What skills can I develop? overview
  • Digital, technology use and technical skills
  • Entrepreneurship & Enterprise
  • Interpersonal, communication and social skills
  • Leadership & Management
  • Learning, thinking, and reasoning skills
  • Planning and organisation skills
  • Presentation, negotiation and influencing skills
  • Self-management skills
  • Writing, analytical and reporting skills
  • Development activities overview
  • Academic study
  • Community and Volunteering
  • Entrepreneurship
  • Professional Development
  • Societies & Committees
  • Theatre, Film & Arts
  • Research students
  • Fees and financial assistance overview
  • Financial assistance overview
  • General eligibility principles and guidance
  • Cambridge Bursary Scheme funding overview
  • What you could get
  • Scottish students
  • EU students
  • Clinical medics and vets
  • Independent students
  • Extra scholarships and awards
  • Crane Fund overview
  • Postgraduate Wellbeing Advice
  • Undergraduate Financial Assistance Fund
  • Postgraduate Financial Assistance Fund
  • Realise Financial Assistance Fund
  • Loan Fund I
  • External Support 
  • Support from your Funding Sponsor
  • Guidance for Academic Supervisors and College Tutors
  • Fees overview
  • Funding overview
  • Mosley, Worts, and Frere Travel Funds
  • Support for UKRI Studentship Holders
  • Student loans overview
  • US loans overview
  • Application procedure
  • Entrance and Exit Counselling
  • Cost of attendance
  • What type of loan and how much you can borrow
  • Interest rates for federal student loans
  • Proof of funding for visa purposes
  • Disbursement
  • Satisfactory academic progress policy
  • In-School Deferment Forms
  • Leave of absence
  • Withdrawing and return to Title IV policy
  • Rights and Responsibilities as a Borrower
  • Managing Repayment
  • Consumer information
  • Submitting a thesis — information for PhD students
  • Private loans
  • Veteran affairs benefits
  • Frequently Asked Questions
  • Student support

Word limits and requirements of your Degree Committee

Candidates should write as concisely as is possible, with clear and adequate exposition. Each Degree Committee has prescribed the limits of length and stylistic requirements as given below. On submission of the thesis you must include a statement of length confirming that it does not exceed the word limit for your Degree Committee.

These limits and requirements are strictly observed by the Postgraduate Committee and the Degree Committees and, unless approval to exceed the prescribed limit has been obtained beforehand (see: Extending the Word Limit below), a thesis that exceeds the limit may not be examined until its length complies with the prescribed limit.

Extending the Word Limit

Thesis word limits are set by Degree Committees. If candidates need to increase their word limits they will need to apply for permission.

Information on how to apply (via self-service account) is available on the ‘ Applying for a change in your student status’  page. If following your viva, you are required to make corrections to your thesis which will mean you need to increase your word-limit, you need to apply for permission in the same way.

Requirements of the Degree Committees

Archaeology and anthropology, architecture and history of art, asian and middle eastern studies, business and management, clinical medicine and clinical veterinary medicine, computer laboratory, earth sciences and geography, scott polar institute, engineering, history and philosophy of science, land economy, mathematics, modern and medieval languages and linguistics, physics and chemistry, politics and international studies, archaeology and social anthropology.

The thesis is not to exceed 80,000 words (approx. 350 pages) for the PhD degree and 60,000 words for the MSc or MLitt degree. These limits include all text, figures, tables and photographs, but exclude the bibliography, cited references and appendices. More detailed specifications should be obtained from the Division concerned. Permission to exceed these limits will be granted only after a special application to the Degree Committee. The application must explain in detail the reasons why an extension is being sought and the nature of the additional material, and must be supported by a reasoned case from the supervisor containing a recommendation that a candidate should be allowed to exceed the word limit by a specified number of words. Such permission will be granted only under exceptional circumstances. If candidates need to apply for permission to exceed the word limit, they should do so in good time before the date on which a candidate proposes to submit the thesis, by application made to the Graduate Committee.

Biological Anthropology:

Students may choose between two alternative thesis formats for their work:

either in the form of a thesis of not more than 80,000 words in length for the PhD degree and 60,000 words for the MSc or MLitt degree. The limits include all text, in-text citations, figures, tables, captions and footnotes but exclude bibliography and appendices; or

in the form of a collection of at least three research articles for the PhD degree and two research articles for the MSc or MLitt degree, formatted as an integrated piece of research, with a table of contents, one or more chapters that outline the scope and provide an in-depth review of the subject of study, a concluding chapter discussing the findings and contribution to the field, and a consolidated bibliography. The articles may be in preparation, submitted for publication or already published, and the combined work should not exceed 80,000 words in length for the PhD degree and 60,000 words for the MSc or MLitt degree. The word limits include all text, in-text citations, figures, tables, captions, and footnotes but exclude bibliography and appendices containing supplementary information associated with the articles. More information on the inclusion of material published, in press or in preparation in a PhD thesis may be found in the Department’s PhD submission guidelines.

Architecture:

The thesis is not to exceed 80,000 words for the PhD and 60,000 words for the MSc or MLitt degree. Footnotes, references and text within tables are to be counted within the word-limit, but captions, appendices and bibliographies are excluded. Appendices should be confined to such items as catalogues, original texts, translations of texts, transcriptions of interview, or tables.

History of Art:

The thesis is not to exceed 80,000 words for the PhD and 60,000 words for the MLitt degree. To include: footnotes, table of contents and list of illustrations, but excluding acknowledgements and the bibliography. Appendices (of no determined word length) may be permitted subject to the approval of the candidate's Supervisor (in consultation with the Degree Committee); for example, where a catalogue of works or the transcription of extensive primary source material is germane to the work. Permission to include such appendices must be requested from the candidate's Supervisor well in advance of the submission of the final thesis. NB: Permission for extensions to the word limit for most other purposes is likely to be refused.

The thesis for the PhD degree is not to exceed 80,000 words exclusive of footnotes, appendices and bibliography but subject to an overall word limit of 100,000 words exclusive of bibliography. Students may apply to include appendices that are exclusive of the word count. In order to be eligible, appendices must be of a none-discursive nature such as lists of linguistic examples, transcriptions of excerpts from works, interviews, inscriptions, archival documents, and critical editions. Each application is subject to the approval of the Degree Committee. For the MLitt degree not to exceed 60,000 words inclusive of footnotes but exclusive of bibliography and appendices.

The thesis for the PhD is not to exceed 60,000 words in length (80,000 by special permission), exclusive of tables, footnotes, bibliography, and appendices. Double-spaced or one-and-a-half spaced. Single or double-sided printing.

The thesis for the MPhil in Biological Science is not to exceed 20,000 words in length, exclusive of tables, footnotes, bibliography, and appendices. Double-spaced or one-and-a-half spaced. Single or double-sided printing.

For the PhD Degree the thesis is not to exceed 80,000 words, EXCLUDING bibliography, but including tables, tables of contents, footnotes and appendices. It is normally expected to exceed 40,000 words unless prior permission is obtained from the Degree Committee. Each page of statistical tables, charts or diagrams shall be regarded as equivalent to a page of text of the same size. The Degree Committee do not consider applications to extend this word limit.

For the Doctor of Business (BusD) the thesis will be approximately 200 pages (a maximum length of 80,000 words, EXCLUDING bibliography, but including tables, tables of contents, footnotes and appendices).

For the MSc Degree the thesis is not to exceed 40,000 words, EXCLUDING bibliography, but including tables, tables of contents, footnotes and appendices.

The thesis is not to exceed 80,000 words including footnotes, references, and appendices but excluding bibliography; a page of statistics shall be regarded as the equivalent of 150 words. Also excluded are title-page, acknowledgements, table of contents, preface, and abstract/summary of thesis. Only under exceptional circumstances will permission be granted to exceed this limit. Candidates must submit with the thesis a signed statement giving the length of the thesis.

For the PhD degree, not to exceed 60,000 words (or 80,000 by special permission of the Degree Committee), and for the MSc degree, not to exceed 40,000 words. These limits exclude figures, photographs, tables, appendices and bibliography. Lines to be double or one-and-a-half spaced; pages to be double or single sided.

The thesis is not to exceed, without the prior permission of the Degree Committee, 60,000 words including tables, footnotes and equations, but excluding appendices, bibliography, photographs and diagrams. Any thesis which without prior permission of the Degree Committee exceeds the permitted limit will be referred back to the candidate before being forwarded to the examiners.

The thesis is not to exceed 80,000 words for the PhD degree and the MLitt degree, including footnotes, references and appendices but excluding bibliography. Candidates must submit with the thesis a signed statement giving the length of the thesis. Only under exceptional circumstances will permission be granted to exceed this limit for the inclusion of an appendix of a substantial quantity of text which is necessary for the understanding of the thesis (e.g. texts in translation, transcription of extensive primary source material). Permission must be sought at least three months before submission of the thesis and be supported by a letter from the supervisor certifying that such exemption from the prescribed limit of length is absolutely necessary.

The thesis is not to exceed, without the prior permission of the Degree Committee, 80,000 words for the PhD degree and 60,000 words for the MSc or MLitt degree, including the summary/abstract.  The table of contents, photographs, diagrams, figure captions, appendices, bibliography and acknowledgements to not count towards the word limit. Footnotes are not included in the word limit where they are a necessary part of the referencing system used.

Earth Sciences:

The thesis is not to exceed, without the prior permission of the Degree Committee, 275 numbered pages of which not more than 225 pages are text, appendices, illustrations and bibliography. A page of text is A4 one-and-a-half-spaced normal size type. The additional 50 pages may comprise tables of data and/or computer programmes reduced in size.

If a candidate's work falls within the social sciences, candidates are expected to observe the limit described in the Department of Geography above; if, however, a candidate's work falls within the natural sciences, a candidate should observe the limit described in the Department of Earth Sciences.

Applications for the limit of length of the thesis to be exceeded must be early — certainly no later than the time when the application for the appointment of examiners and the approval of the title of the thesis is made. Any thesis which, without the prior permission of the Degree Committee, exceeds the permitted limit of length will be referred back to the candidate before being forwarded to the examiners.

The thesis is not to exceed, without the prior permission of the Degree Committee, 60,000 words including tables, footnotes, bibliography and appendices. The Degree Committee points out that some of the best thesis extend to only half this length. Each page of statistical tables, charts or diagrams shall be regarded as equivalent to a page of text of the same size.

The thesis is not to exceed 80,000 words for the PhD and EdD degrees and 60,000 words for the MSc and MLitt degrees, in all cases excluding appendices, footnotes, reference list or bibliography. Only in the most exceptional circumstances will permission be given to exceed the stated limits. In such cases, you must make an application to the Degree Committee as early as possible -and no later than three months before it is proposed to submit the thesis, having regard to the dates of the Degree Committee meetings. Your application should (a) explain in detail the reasons why you are seeking the extension and (b) be accompanied by a full supporting statement from your supervisor showing that the extension is absolutely necessary in the interests of the total presentation of the subject.

For the PhD degree, not to exceed, without prior permission of the Degree Committee, 65,000 words, including appendices, footnotes, tables and equations not to contain more than 150 figures, but excluding the bibliography. A candidate must submit with their thesis a statement signed by the candidate themself giving the length of the thesis and the number of figures. Any thesis which, without the prior permission of the Degree Committee, exceeds the permitted limit will be referred back to the candidate before being forwarded to the examiners.

The thesis is not to exceed 80,000 words or go below 60,000 words for the PhD degree and not to exceed 60,000 words or go below 45,000 words for the MLitt degree, both including all notes and appendices but excluding the bibliography. A candidate must add to the preface of the thesis the following signed statement: 'The thesis does not exceed the regulation length, including footnotes, references and appendices but excluding the bibliography.'

In exceptional cases (when, for example, a candidate's thesis largely consists of an edition of a text) the Degree Committee may grant permission to exceed these limits but in such instances (a) a candidate must apply to exceed the length at least three months before the date on which a candidate proposes to submit their thesis and (b) the application must be supported by a letter from a candidate's supervisor certifying that such exemption from the prescribed limit of length is absolutely necessary.

It is a requirement of the Degree Committee for the Faculty of English that thesis must conform to either the MHRA Style Book or the MLA Handbook for the Writers of Research papers, available from major bookshops. There is one proviso, however, to the use of these manuals: the Faculty does not normally recommend that students use the author/date form of citation and recommends that footnotes rather than endnotes be used. Bibliographies and references in thesis presented by candidates in ASNaC should conform with either of the above or to the practice specified in Cambridge Studies in Anglo-Saxon England.

Thesis presented by candidates in the Research Centre for English and Applied Linguistics must follow as closely as possible the printed style of the journal Applied Linguistics and referencing and spelling conventions should be consistent.

A signed declaration of the style-sheet used (and the edition, if relevant) must be made in the preliminary pages of the thesis.

PhD theses MUST NOT exceed 80,000 words, and will normally be near that length.

A minimum word length exists for PhD theses: 70,000 words (50,000 for MLitt theses)

The word limit includes appendices and the contents page but excludes the abstract, acknowledgments, footnotes, references, notes on transliteration, bibliography, abbreviations and glossary.  The Contents Page should be included in the word limit. Statistical tables should be counted as 150 words per table. Maps, illustrations and other pictorial images count as 0 words. Graphs, if they are the only representation of the data being presented, are to be counted as 150 words. However, if graphs are used as an illustration of statistical data that is also presented elsewhere within the thesis (as a table for instance), then the graphs count as 0 words.

Only under exceptional circumstances will permission be granted to exceed this limit. Applications for permission are made via CamSIS self-service pages. Applications must be made at least four months before the thesis is bound. Exceptions are granted when a compelling intellectual case is made.

The thesis is not to exceed 80,000 words for the PhD degree and 60,000 words for the MLitt degree, in all cases including footnotes and appendices but excluding bibliography. Permission to submit a thesis falling outside these limits, or to submit an appendix which does not count towards the word limit, must be obtained in advance from the Degree Committee.

The thesis is not to exceed 80,000 for the PhD degree and 60,000 words for the MSc or MLitt degree, both including footnotes, references and appendices but excluding bibliographies. One A4 page consisting largely of statistics, symbols or figures shall be regarded as the equivalent of 250 words. A candidate must add to the preface of their thesis the following signed statement: 'This thesis does not exceed the regulation length, including footnotes, references and appendices.'

For the PhD degree the thesis is not to exceed 80,000 words (exclusive of footnotes, appendices and bibliography) but subject to an overall word limit of 100,000 words (exclusive of bibliography, table of contents and any other preliminary matter). Figures, tables, images etc should be counted as the equivalent of 200 words for each A4 page, or part of an A4 page, that they occupy. For the MLitt degree the thesis is not to exceed 60,000 words inclusive of footnotes but exclusive of bibliography, appendices, table of contents and any other preliminary matter. Figures, tables, images etc should be counted as the equivalent of 200 words for each A4 page, or part of an A4 page, that they occupy.

Criminology:

For the PhD degree submission of a thesis between 55,000 and 80,000 words (exclusive of footnotes, appendices and bibliography) but subject to an overall word limit of 100,000 words (exclusive of bibliography, table of contents and any other preliminary matter). Figures, tables, images etc should be counted as the equivalent of 200 words for each A4 page, or part of an A4 page, that they occupy. For the MLitt degree the thesis is not to exceed 60,000 words inclusive of footnotes but exclusive of bibliography, appendices, table of contents and any other preliminary matter. Figures, tables, images etc should be counted as the equivalent of 200 words for each A4 page, or part of an A4 page, that they occupy.

There is no standard format for the thesis in Mathematics.  Candidates should discuss the format appropriate to their topic with their supervisor.

The thesis is not to exceed 80,000 words for the PhD degree and 60,000 words for the MLitt degree, including footnotes and appendices but excluding the abstract, any acknowledgements, contents page(s), abbreviations, notes on transliteration, figures, tables and bibliography. Brief labels accompanying illustrations, figures and tables are also excluded from the word count. The Degree Committee point out that some very successful doctoral theses have been submitted which extend to no more than three-quarters of the maximum permitted length.

In linguistics, where examples are cited in a language other than Modern English, only the examples themselves will be taken into account for the purposes of the word limit. Any English translations and associated linguistic glosses will be excluded from the word count.

In theses written under the aegis of any of the language sections, all sources in the language(s) of the primary area(s) of research of the thesis will normally be in the original language. An English translation should be provided only where reading the original language is likely to fall outside the expertise of the examiners. Where such an English translation is given it will not be included in the word count. In fields where the normal practice is to quote in English in the main text, candidates should follow that practice. If the original text needs to be supplied, it should be placed in a footnote. These fields include, but are not limited to, general linguistics and film and screen studies.

Since appendices are included in the word limit, in some fields it may be necessary to apply to exceed the limit in order to include primary data or other materials which should be available to the examiners. Only under the most exceptional circumstances will permission be granted to exceed the limit in other cases. In all cases (a) a candidate must apply to exceed the prescribed maximum length at least three months before the date on which a candidate proposes to submit their thesis and (b) the application must be accompanied by a full supporting statement from the candidate's supervisor showing that such exemption from the prescribed limit of length is absolutely necessary.

It is a requirement within all language sections of MMLL, and also for Film, that dissertations must conform with the advice concerning abbreviations, quotations, footnotes, references etc published in the Style Book of the Modern Humanities Research Association (Notes for Authors and Editors). For linguistics, dissertations must conform with one of the widely accepted style formats in their field of research, for example the style format of the Journal of Linguistics (Linguistic Association of Great Britain), or of Language Linguistic Society of America) or the APA format (American Psychology Association). If in doubt, linguistics students should discuss this with their supervisor and the PhD Coordinator.

The thesis is not to exceed 80,000 words for the PhD degree and 60,000 words for the MLitt degree, both excluding notes, appendices, and bibliographies, musical transcriptions and examples, unless a candidate make a special case for greater length to the satisfaction of the Degree Committee. Candidates whose work is practice-based may include as part of the doctoral submission either a portfolio of substantial musical compositions, or one or more recordings of their own musical performance(s).

PhD (MLitt) theses in Philosophy must not be more than 80,000 (60,000) words, including appendices and footnotes but excluding bibliography.

Institute of Astronomy, Department of Materials Science & Metallurgy, Department of Physics:

The thesis is not to exceed, without prior permission of the Degree Committee, 60,000 words, including summary/abstract, tables, footnotes and appendices, but excluding table of contents, photographs, diagrams, figure captions, list of figures/diagrams, list of abbreviations/acronyms, bibliography and acknowledgements.

Department of Chemistry:

The thesis is not to exceed, without prior permission of the Degree Committee, 60,000 words, including summary/abstract, tables, and footnotes, but excluding table of contents, photographs, diagrams, figure captions, list of figures/diagrams, list of abbreviations/acronyms, bibliography, appendices and acknowledgements. Appendices are relevant to the material contained within the thesis but do not form part of the connected argument. Specifically, they may include derivations, code and spectra, as well as experimental information (compound name, structure, method of formation and data) for non-key molecules made during the PhD studies.

Applicable to the PhDs in Politics & International Studies, Latin American Studies, Multi-disciplinary Studies and Development Studies for all submissions from candidates admitted prior to and including October 2017.

A PhD thesis must not exceed 80,000 words, and will normally be near that length. The word limit includes appendices but excludes footnotes, references and bibliography. Footnotes should not exceed 20% of the thesis. Discursive footnotes are generally discouraged, and under no circumstances should footnotes be used to include material that would normally be in the main text, and thus to circumvent the word limits. Statistical tables should be counted as 150 words per table. Only under exceptional circumstances, and after prior application, will the Degree Committee allow a student to exceed these limits. A candidate must submit, with the thesis, a statement signed by her or himself attesting to the length of the thesis. Any thesis that exceeds the limit will be referred back to candidate for revision before being forwarded to the examiners.

Applicable to the PhDs in Politics & International Studies, Latin American Studies, Multi-disciplinary Studies and Development Studies for all submissions from candidates admitted after October 2017.

A PhD thesis must not exceed 80,000 words, including footnotes. The word limit includes appendices but excludes the bibliography. Discursive footnotes are generally discouraged, and under no circumstances should footnotes be used to include material that would normally be in the main text. Statistical tables should be counted as 150 words per table. Only under exceptional circumstances, and after prior application, will the Degree Committee allow a student to exceed these limits. A candidate must submit, with the thesis, a statement signed by her or himself attesting to the length of the thesis. Any thesis that exceeds the limit will be referred back to candidate for revision before being forwarded to the examiners.

Only applicable to students registered for the degree prior to 1 August 2012; all other students should consult the guidance of the Faculty of Biological Sciences.

Applicable to the PhD in Psychology (former SDP students only) for all submissions made before 30 November 2013

A PhD thesis must not exceed 80,000 words, and will normally be near that length. The word limit includes appendices but excludes footnotes, references and bibliography. Footnotes should not exceed 20% of the thesis. Discursive footnotes are generally discouraged, and under no circumstances should footnotes be used to include material that would normally be in the main text, and thus to circumvent the word limits. Statistical tables should be counted as 150 words per table. Only under exceptional circumstances, and after prior application, will the Degree Committee allow a student to exceed these limits. A candidate must submit, with the thesis, a statement signed by her or himself attesting to the length of the thesis. Any thesis that exceeds the limit will be referred back to candidate for revision before being forwarded to the examiners.

Applicable to the PhD in Psychology (former SDP students only) for all submissions from 30 November 2013

A PhD thesis must not exceed 80,000 words, and will normally be near that length. The word limit includes appendices but excludes footnotes, references and bibliography. Footnotes should not exceed 20% of the thesis. Discursive footnotes are generally discouraged, and under no circumstances should footnotes be used to include material that would normally be in the main text, and thus to circumvent the word limits. Statistical tables should be counted as 150 words per table. Only under exceptional circumstances, and after prior application, will the Degree Committee allow a student to exceed these limits. Applications should be made in good time before the date on which a candidate proposes to submit the thesis, made to the Graduate Committee. A candidate must submit, with the thesis, a statement signed by her or himself attesting to the length of the thesis. Any thesis that exceeds the limit will be referred back to candidate for revision before being forwarded to the examiners.

A PhD thesis must not exceed 80,000 words, and will normally be over 60,000 words. This word limit includes footnotes and endnotes, but excludes appendices and reference list / bibliography. Figures, tables, images etc should be counted as the equivalent of 150 words for each page, or part of a page, that they occupy. Other media may form part of the thesis by prior arrangement with the Degree Committee. Students may apply to the Degree Committee for permission to exceed the word limit, but such applications are granted only rarely. Candidates must submit, with the thesis, a signed statement attesting to the length of the thesis.

© 2024 University of Cambridge

  • Contact the University
  • Accessibility
  • Freedom of information
  • Privacy policy and cookies
  • Statement on Modern Slavery
  • Terms and conditions
  • University A-Z
  • Undergraduate
  • Postgraduate
  • Research news
  • About research at Cambridge
  • Spotlight on...

Academia Insider

How long is a PhD dissertation? [Data by field]

The final piece of the PhD journey is the PhD dissertation. It takes many years to accumulate enough original and new data to fill out a dissertation to the satisfaction of experts in your field. Interestingly, the PhD dissertation length and content vary significantly based on the field you are studying and the publishing conventions.

A PhD can be anywhere from 50 pages to over 450 pages long. This equates to between about 20,000 words to 100,000 words. Most PhD theses are between 60,000 and 80,000 words long excluding contents, citations and references.

A PhD thesis contains different sections including an introduction, methods, results and discussion, conclusions, further work, and references. Each one of these different sections will vary in length depending on the field of study and your particular topic.

Ultimately, a PhD dissertation should contain as many pages and words as it takes to communicate the results of your multi-year investigation.

It is very rewarding to see your thesis come together as you are writing day after day. When I was writing my PhD dissertation I wrote the sections separately and my heart filled with joy when I finally put them all together and compile them into a single PDF document.

Counting the pages should not be the way to determine a PhD dissertation’s value but it certainly helps when your thesis is starting to look substantial in thickness.

How many pages should a PhD dissertation be?

A PhD dissertation should contain as many pages and words as it takes to outline the current state of your field and provide adequate background information, present your results, and provide confidence in your conclusions. A PhD dissertation will also contain figures, graphs, schematics, and other large pictorial items that can easily inflate the page count.

Here is a boxplot summary of many different fields of study and the number of pages of a typical PhD dissertation in the field. It has been created by Marcus Beck from all of the dissertations at the University of Minnesota.

biology phd thesis length

Typically, the mathematical sciences, economics, and biostatistics theses and dissertations tend to be shorter because they rely on mathematical formulas to provide proof of their results rather than diagrams and long explanations.

On the other end of the scale, English, communication studies, political science, history and anthropology are often the largest theses in terms of pages and word count because of the number of words it takes to provide proof and depth of their results.

At the end of the day, it is important that your thesis gets signed off by your review committee and other experts in the field. Your supervisor will be the main judge of whether or not your dissertation is capable of satisfying the requirements of a PhD in your field.

If you want to know more about how long a Masters’s thesis and PhD dissertation is you can check out my other articles:

  • How Long is a Masters Thesis? [Your writing guide]
  • How long is a Thesis or dissertation? [the data]

Can a PhD dissertation be too long?

A PhD thesis should contain enough evidence and discussion to report on the most significant findings of your PhD research.

A PhD dissertation should not contain everything that you have done during your PhD. It should only include the data and information required to convince your PhD examining body that wraps up and tells the full story of particular lines of investigation.

Including random results, thoughts, or superfluous explanation can result in a dissertation that is unfocused. I have heard of music PhD is being described as too verbose and physical sciences PhD dissertations as being unfocused.

Therefore, a PhD thesis can be too long if the information it contains does not form a full and cohesive story.

One of my colleagues during their PhD removed an entire chapter from the thesis after writing it as the supervisor said that it needed more experiments to be a full story. They did not want to spend the next six months gathering the data and simply removed the chapter altogether.

How short can PhD dissertation be?

The shortest PhD dissertations are typically found in mathematics.

George Bernard Danzig was an American mathematical scientist who made contributions to industrial engineering and many other mathematical-related fields. An interesting miscommunication led to 1 of the shortest PhD theses ever.

In 1939 his professor wrote two problems on the blackboard and Danzig thought they were homeless assignments. He stated that they were harder than usual but handed in solutions to the surprise of the professor.

They were, in fact, open mathematical problems in statistics.

His professor said to bind the solution to the two problems together and submit them as his thesis – the total thesis length = 14 pages.

Obviously, most PhD theses and dissertations will be so much longer than that!

My PhD dissertation was 256 pages long. It was full of schematics, diagrams, and tables to demonstrate and communicate my findings.

I would say that most people’s PhD thesis experience will be closer to mine than Prof George Bernard Danzig’s.

Why PhD dissertations are typically so long

PhD dissertations are often over 200 pages long.

One of the primary reasons they are so long is that it is a single document that summarises many years of hard work. Also, summarising the research field to date and making sure that all of your references and citations are included so you avoid plagiarism will bolster the word count of the thesis dramatically.

Here are all of the reasons PhD dissertations tend to be so long.

Many years of work

PhD theses or dissertations contain many years of research and analysis.

In many of my YouTube videos I recommend that a PhD student work towards their PhD thesis by doing at least three hours of focused work every work day.

This amount of work quickly adds up.

Of course, not every bit of work makes it into the PhD dissertation but a lot of it does. It can be difficult to work out what to include or leave out of your thesis.

As a PhD student, I perfected the art of turning one experiment into many different types of grafts and schematics to fully explore the limits of my data. The graphs can take up a lot of space in your PhD thesis and, therefore, bolster the page count significantly.

In depth literature review

One of the most substantial parts of a PhD dissertation is the literature review.

The literature review can take up a huge portion of the early part of your PhD dissertation depending on the amount of data and publications in your field.

Writing an in-depth literature review requires just as much meticulous data analysis and searching as the central part of your dissertation.

Figures and schematics

Some fields end up producing a lot of figures and schematics.

My thesis had many full-page figures of atomic force microscopy experiments with much more explanation on subsequent pages.

biology phd thesis length

As they say, a picture paints a thousand words and a dissertation can really benefit from having many schematics to highlight the important aspects of your findings.

References and citations

The recommended PhD dissertation word count from an institution or university does not include citations, references, or other thesis parts such as summary of abbreviations, table of figures, et cetera.

However, these components of your dissertation can take up many pages and add to the overall thickness of your PhD dissertation.

University formatting rules

University formatting rules will also dictate how you many pages your words take up.

I often get roasted on my YouTube channel for having doublespaced lines and wide margins. Unfortunately, this layout was dictated by my university before printing.

PhD dissertations often end up going into long-term storage and therefore, need to adhere to archival and standardised formatting rules.

Deep in the depths of the University of Newcastle, there is a copy of my thesis on a shelf. The formatting and binding rules mean that my thesis looks like everyone else’s.

Universities will often have their own requirements for PhD dissertation cover colour, quality, and type of paper. Even the quality of the paper can change the thickness of the PhD dissertation significantly.

PhD by publication

It is becoming increasingly common to submit a number of peer-reviewed papers bound together with supplementary information in between instead of a PhD dissertation.

The benefits of this to the researcher and university are:

  • More early career peer-reviewed journals for career advancement
  • an easier review process – they have already been peer-reviewed
  • an early focus on publishing means better research outcomes for the researcher, supervisor, and Department.
  • No mad rush at the end to finish a thesis
  • continually writing peer-reviewed papers throughout your PhD helps with timely analysis and communication of results

Even though this option has been available to PhD students for a number of years, I have only known a handful of students actually submit their PhD via publication.

Nonetheless, having this option will suit some research fields better than others and lead to a more productive PhD.

Wrapping up

This article has been through everything you need to know about the length of a PhD dissertation and the common lengths of PhD dissertations for various fields.

Ultimately, there is no predefined length of a PhD .

A PhD thesis is as long as it needs to be to convince your examiners that you have contributed significantly enough to an academic field to be awarded the title of Dr of philosophy.

Mathematical and analytical theses tend to be shorter and can be as short as 50 pages (with one of the shortest being only 14 pages long). At the other end of the spectrum, PhD students in anthropology and history tend to produce the longest dissertations.

biology phd thesis length

Dr Andrew Stapleton has a Masters and PhD in Chemistry from the UK and Australia. He has many years of research experience and has worked as a Postdoctoral Fellow and Associate at a number of Universities. Although having secured funding for his own research, he left academia to help others with his YouTube channel all about the inner workings of academia and how to make it work for you.

Thank you for visiting Academia Insider.

We are here to help you navigate Academia as painlessly as possible. We are supported by our readers and by visiting you are helping us earn a small amount through ads and affiliate revenue - Thank you!

biology phd thesis length

2024 © Academia Insider

biology phd thesis length

biology phd thesis length

Research Voyage

Research Tips and Infromation

How Long Should Be PhD Dissertation? Unlocking The Mystery of PhD Thesis Length

PhD Dissertation length

Embarking on the journey of a PhD is a scholarly endeavour that demands not only intellectual prowess but also a deep commitment to contributing valuable insights to the academic world. At the heart of this rigorous pursuit lies the dissertation, a magnum opus that serves as the pinnacle of one’s academic achievement. Yet, as aspiring scholars delve into the realms of research and knowledge creation, a pivotal question looms large: How long should a Ph.D. dissertation be?

The optimal length of a Ph.D. dissertation is a strategic equilibrium, determined by the intricate interplay of research complexity, disciplinary norms, and institutional guidelines, emphasizing quality over mere quantity.

In the intricate tapestry of academia, the length of a dissertation is a nuanced consideration, influenced by a myriad of factors ranging from disciplinary norms to the intricacies of research design. In this exploration, we embark on a journey to unravel the complexities surrounding dissertation length, understanding the guiding principles, and offering insights into how aspiring doctoral candidates can strike the delicate balance between depth and brevity.

Join us as we navigate the academic landscape, demystifying the expectations, uncovering the variances across disciplines, and providing practical tips for crafting a dissertation that stands as a testament to scholarly excellence. Whether you’re at the threshold of your doctoral journey or guiding others through its twists and turns, this discourse aims to shed light on the intricacies of dissertation length, empowering you to embark on this intellectual odyssey with confidence and purpose.

Introduction

A. defining the purpose.

  • B. The Dissertation's Role in Academic Knowledge

C. Emphasizing Thorough Research and Original Contributions

A. exploring institutional guidelines, b. specific requirements and length expectations, c. the importance of adherence, a. acknowledging disciplinary differences, b. examples of varied expectations, c. emphasizing disciplinary norms, a. research complexity, b. data collection and analysis, c. literature review, d. methodology, a. emphasizing the importance of balance, b. strategies for maintaining focus and relevance, c. encouraging quality over quantity, a. effective time management, b. techniques for concise writing, c. value of feedback from advisors and peers.

Embarking on a PhD journey is akin to setting sail into uncharted waters of knowledge, with the dissertation standing tall as the crowning achievement. It’s not just a document; it’s a testament to years of intellectual toil, research finesse, and a contribution to the vast expanse of human understanding. But as scholars immerse themselves in the sea of research, a question inevitably surfaces: How long should a Ph.D. dissertation be?

Significance Unveiled

A Ph.D. dissertation is more than a lengthy document; it’s a scholarly masterpiece that not only encapsulates an individual’s intellectual prowess but adds a unique brushstroke to the grand canvas of academia. It’s a beacon of knowledge, guiding future researchers and shaping the trajectory of scholarly discourse. Understanding its significance is the first step toward unlocking the secrets of its length.

Navigating Length: A Delicate Balancing Act

Determining the appropriate length for a Ph.D. dissertation is no mere formality. It’s a delicate dance between providing comprehensive insights and maintaining reader engagement. Striking this balance is crucial, as the length not only influences how the research is perceived but also reflects the researcher’s ability to weave a compelling narrative without unnecessary verbosity.

Factors in the Equation

The length of a dissertation is a dynamic variable influenced by a multitude of factors. From the complexity of the research question to the intricacies of data collection, each element plays a role. It’s a symphony where the depth of the literature review, the intricacy of the methodology, and the nature of the chosen discipline all contribute to the final crescendo of the dissertation length.

Purpose of a PhD Dissertation

At the heart of the Ph.D. journey lies the dissertation, a formidable endeavour that goes beyond being a mere academic requirement. It serves as the cornerstone of a scholar’s intellectual legacy, encapsulating years of research, critical thinking, and a commitment to advancing knowledge in a specific field. The purpose of a Ph.D. dissertation extends far beyond a graduation requirement; it is a scholarly rite of passage that marks one’s entry into the ranks of contributing intellectuals.

B. The Dissertation’s Role in Academic Knowledge

The dissertation, in essence, is a torchbearer of academic knowledge. It doesn’t merely regurgitate existing information but actively contributes to the ongoing conversation within a field. Picture it as a puzzle piece that, when seamlessly integrated, enriches the larger mosaic of human understanding. Its role is not just to summarize what is known but to illuminate uncharted territories, challenging established paradigms and paving the way for new insights.

Example: In the field of environmental science, a Ph.D. dissertation might delve into the impact of climate change on a specific ecosystem, offering novel findings that reshape our understanding of ecological resilience. It becomes a pivotal contribution that informs future research and policy decisions.

Thorough research is the bedrock upon which a meaningful dissertation stands. It involves meticulously exploring existing literature, methodologies, and gaps in knowledge. This isn’t a cursory glance but a deep dive into the scholarly ocean, where each wave of information contributes to the construction of a comprehensive understanding.

Original contributions are the soul of a Ph.D. dissertation. It’s not just about rehashing what’s already known but about introducing something new and transformative. This could be a novel research methodology, a groundbreaking theory, or empirical findings that challenge existing theories.

Example: In the realm of psychology, a Ph.D. dissertation might involve the development of a new therapeutic approach for a specific mental health condition, backed by both a comprehensive review of existing literature and empirical evidence from original studies. This not only adds to the academic discourse but also has tangible implications for clinical practice.

University Guidelines PhD Dissertation Length: Navigating the Academic Framework

In academia, each university sets the stage with its own set of guidelines governing the composition and expectations of a Ph.D. dissertation. These guidelines serve as the rulebook, providing a roadmap for aspiring scholars to traverse the challenging terrain of research and writing. Understanding these guidelines is akin to deciphering the code that unlocks the door to doctoral success.

Dive into the particulars, and you’ll find that universities often outline specific requirements and expectations regarding the length of a dissertation. These may range from prescribed word counts for each section to broader expectations for the overall document. Some institutions might emphasize brevity, while others encourage a more expansive exploration. Unravelling these expectations is not just a bureaucratic formality; it’s a strategic move that ensures your work aligns with the academic standards set by your institution.

Example: Imagine a university that places a strong emphasis on concise and focused dissertations. Here, the guidelines may state a maximum word count for each chapter, encouraging a streamlined and impactful presentation of research findings. Adhering to these specifics ensures that your dissertation not only meets the academic standards but also resonates with the evaluators who appreciate clarity and precision.

While the allure of academic freedom may tempt scholars to chart their own course, adherence to institutional guidelines is paramount. It’s not just a matter of compliance; it’s a strategic decision that aligns your work with the expectations of the academic community. Universities set guidelines with a purpose – to maintain standards, ensure consistency, and facilitate fair evaluation. Ignoring these guidelines can inadvertently hinder the reception of your dissertation, potentially overshadowing the brilliance of your research with concerns about adherence to academic norms.

Disciplinary Variances: Navigating the Length Spectrum

In the vast landscape of academia, one size certainly does not fit all, especially when it comes to the length of Ph.D. dissertations. It’s crucial to acknowledge and appreciate the dynamic nature of disciplinary differences, where each field has its own set of expectations, traditions, and scholarly norms. Understanding this diversity is the key to crafting a dissertation that resonates within the specific academic community to which it belongs.

Let’s take a stroll through different academic landscapes to grasp the breadth of expectations. In the sciences, precision and brevity often reign supreme. A molecular biology dissertation, for instance, may prioritize concise methodology and results chapters, with an emphasis on data interpretation. Contrast this with a dissertation in the humanities, where the richness of language and the depth of literary analysis might lead to a more extensive exploration of concepts.

Example: In engineering, a Ph.D. dissertation might lean towards a succinct presentation of methodologies, experimental results, and their implications for the field. Meanwhile, in the realm of philosophy, a dissertation could be characterized by a more expansive engagement with existing literature and a thorough philosophical exploration of the research question.

The beauty of academia lies in its diversity, but this very diversity requires scholars to be attuned to the norms of their specific discipline. What might be considered a concise and impactful dissertation in one field could be seen as lacking depth in another. Recognizing these norms is not just a matter of fitting in; it’s a strategic decision that ensures your work aligns with the expectations of your academic peers.

Understanding disciplinary norms is like speaking the language of your scholarly community. It’s about knowing when to be succinct and when to elaborate, when to prioritize methodology and when to delve deep into theoretical frameworks.

Factors Influencing PhD Dissertation Length

1. Impact of Research Question Complexity

The complexity of your research question is like the compass guiding the depth and breadth of your dissertation. Intricate inquiries often demand more comprehensive exploration, delving into multiple facets and dimensions. For instance, a research question investigating the genetic determinants of a rare disease may necessitate an extensive review of existing literature, detailed methodologies, and intricate analyses to uncover meaningful insights.

2. Examples of Research Topics

Consider the following examples to illustrate the point:

  • Less Complex: An analysis of consumer behavior in response to a specific marketing strategy might require a detailed but more straightforward exploration.
  • More Complex: On the other hand, a study examining the intersection of artificial intelligence and ethical considerations in healthcare may demand a multifaceted investigation into both technological and ethical dimensions, significantly impacting the length of the dissertation.

1. Influence of Data Nature and Quantity

The nature and amount of data collected cast a profound shadow on dissertation length. A project relying on extensive datasets, intricate statistical analyses, or comprehensive case studies inherently demands a more extended exploration. In contrast, qualitative research might be more concise but equally impactful in unraveling complex phenomena.

2. Role of Statistical Analysis, Case Studies, or Qualitative Research

Consider the following scenarios:

  • Statistical Analysis: A dissertation delving into the economic impact of climate change policies might involve sophisticated statistical models and require an in-depth presentation of results and their implications.
  • Qualitative Research: Conversely, a dissertation employing qualitative interviews to explore the lived experiences of individuals facing a specific social challenge may present findings in a more narrative form.

1. Significance of a Comprehensive Literature Review

A robust literature review acts as the scaffolding for your dissertation, providing the theoretical foundation and context for your research. The broader and more complex the field, the more extensive the literature review. For instance, exploring a niche area within a rapidly evolving field, like emerging technologies, may demand a more thorough literature review to capture the latest developments and debates.

2. Contribution to Overall Length

Consider this:

  • A dissertation in environmental science, investigating the impact of urbanization on biodiversity, might necessitate a detailed exploration of existing literature on ecology, urban planning, and biodiversity conservation.

1. Impact of Detailed Research Methodology

The methodology section is the blueprint of your research, and its level of detail significantly influences the length of your dissertation. A dissertation with a meticulous methodology section is like a well-constructed building, providing a clear roadmap for readers to understand the research process.

2. Need for Clarity and Precision in Research Design

For example:

  • A dissertation in public health aiming to assess the effectiveness of a health intervention might require a detailed explanation of the study design, participant recruitment strategies, and data collection methods to ensure the study’s validity and reliability.

Balancing Depth and Brevity: Crafting a Dissertation Masterpiece

In the symphony of scholarly writing, achieving harmony between depth and brevity is a skill that distinguishes a stellar dissertation. While delving deep into the nuances of your research is essential, presenting it with conciseness ensures that your audience remains engaged. The challenge lies in striking the right balance, where the richness of content is not sacrificed on the altar of brevity.

  • Clear Research Objectives: Begin with well-defined research objectives that serve as the North Star for your dissertation. This clarity guides your writing, preventing unnecessary tangents and ensuring each section contributes directly to your overarching goals. Example: In a dissertation exploring the impact of mindfulness-based interventions on stress reduction, clear objectives would center around understanding the effectiveness of specific mindfulness techniques and their implications for stress management.
  • Thematic Structure: Organize your dissertation thematically, ensuring each chapter has a clear purpose and contributes to building a cohesive narrative. This not only aids readability but also enhances the overall impact of your research. Example: In a literature review, grouping studies thematically—such as by intervention type or outcome measures—provides a structured and focused presentation of existing research, avoiding a scattered and disjointed narrative.
  • Rigorous Editing: Approach your writing with a discerning eye during the editing process. Trim unnecessary words, sentences, or sections that do not directly contribute to the core message of your dissertation. Be ruthless in maintaining relevance. Example: In the methodology section, focus on explaining key decisions and processes, omitting redundant details that do not influence the study’s validity or replicability.
  • Depth of Analysis: Instead of inundating your dissertation with an abundance of superficial analyses, delve deep into a few key points. Thoroughly explore the significance and implications of your findings, providing a nuanced understanding of your research. Example: In a discussion section, rather than covering multiple tangential points, focus on the most critical aspects of your results, analyzing their theoretical and practical implications in detail.
  • Precision in Language: Choose words judiciously to convey your ideas with precision. Aim for clarity without unnecessary embellishments, ensuring that each sentence adds value to your argument. Example: In the introduction, use concise language to clearly articulate the research gap, the significance of the study, and the specific research questions without unnecessary elaboration.
  • Selectivity in Citations: While a comprehensive literature review is vital, selectively cite studies that directly contribute to your research context. Avoid an exhaustive list that overwhelms readers with unnecessary details. Example: In a literature review on educational interventions for students with learning disabilities, selectively cite studies that showcase diverse approaches and methodologies, providing a nuanced understanding of the existing landscape.

As we navigate the delicate equilibrium between depth and brevity, remember that a well-crafted dissertation is not measured by its length but by the impact of its scholarly contributions. Join us as we explore the art of balancing substance with succinctness, ensuring your dissertation stands as a masterpiece in the annals of academic inquiry.

Tips for Managing PhD Dissertation Length: Navigating the Dissertation Writing Process

  • Setting Realistic Milestones: Break down the dissertation-writing process into manageable milestones. Establish realistic timelines for each section, considering the complexity and time required for research, writing, and revisions. Example: Allocate a specific time frame for conducting literature reviews, data analysis, and drafting each chapter. This ensures steady progress and prevents last-minute rushes.
  • Regular Progress Checks: Monitor your progress regularly and adjust your schedule as needed. If a particular section is taking longer than anticipated, evaluate the reasons and recalibrate your timeline accordingly. Example: If data analysis is proving more time-consuming than expected, revisit your research plan and, if necessary, adjust your writing schedule for subsequent chapters.
  • Balancing Research and Writing: Strive for a balance between conducting research and writing. While research is crucial, allocate dedicated time for translating findings into written content to avoid a backlog of information. Example: If you’re conducting experiments, allocate specific writing sessions to summarize and interpret the results, ensuring a continuous flow of progress.

Visit my article on ” How to Manage Research Time” for managing PhD time.

  • Clarity in Expression: Aim for clarity in your writing. Express complex ideas in straightforward language, avoiding unnecessary jargon or convoluted sentences that can inflate the word count without adding substance. Example: Instead of using complex terminology in a theoretical framework, opt for clear and precise language that conveys the theoretical concepts without unnecessary embellishments.
  • Economical Word Choices: Choose words judiciously. Opt for strong, impactful words that convey your message succinctly. Trim redundant phrases and eliminate words that don’t contribute directly to your argument. Example: Instead of saying “due to the fact that,” use the more concise “because” to convey the same meaning with fewer words.
  • Strategic Use of Figures and Tables: Incorporate visuals strategically to convey information efficiently. Figures and tables can often replace lengthy textual explanations, providing a visual representation of data or concepts. Example: Instead of describing a complex set of results in paragraphs, present key findings in a well-designed table, allowing readers to grasp the information at a glance.
  • Early and Regular Feedback: Share drafts of your work with advisors and peers early in the writing process. Their feedback can identify potential issues and guide revisions, preventing the need for extensive rewrites later. Example: Submit a draft of your literature review to your advisor before completing the entire chapter. Early feedback can help refine your approach and ensure you’re on the right track.
  • Objective External Perspectives: Advisors and peers offer valuable external perspectives. They can identify areas where your explanation may be unclear or where additional details may be necessary. Example: If your research methodology is intricate, seek feedback from a peer who is not intimately familiar with your topic. Their questions and comments can reveal where additional clarification is needed for a broader audience.
  • Critical Review for Redundancy: Advisors and peers can help identify redundant sections or unnecessary details. A fresh set of eyes can pinpoint areas where content can be streamlined without compromising the depth of your argument. Example: If two sections of your dissertation cover similar ground, feedback from others can highlight the need to merge or eliminate redundant content, improving the overall flow.

Visit my articles related to PhD , Exciting Careers after PhD .

The journey through the complexities of determining the length of a Ph.D. dissertation reveals a delicate interplay of factors crucial to its scholarly impact. From understanding the significance of this academic endeavour to navigating institutional guidelines and disciplinary variances, the pursuit of balance between depth and brevity emerges as a paramount challenge.

As researchers, we must navigate the intricate landscapes of research complexity, data analysis, literature review, and methodology while maintaining a steadfast commitment to quality over quantity. The tips offered for effective time management, concise writing, and the judicious seeking of feedback underscore the strategic nature of dissertation crafting.

Ultimately, the dissertation is not just an academic requirement but a scholarly legacy—a testament to our intellectual contributions and a beacon guiding future inquiry. In this conclusion, let us recognize that the true measure of a dissertation’s success lies not solely in its length but in its enduring impact on the trajectory of knowledge within our respective fields.

Upcoming Events

  • Visit the Upcoming International Conferences at Exotic Travel Destinations with Travel Plan
  • Visit for  Research Internships Worldwide

Dr. Vijay Rajpurohit

Recent Posts

  • Best 5 Journals for Quick Review and High Impact in August 2024
  • 05 Quick Review, High Impact, Best Research Journals for Submissions for July 2024
  • Top Mistakes to Avoid When Writing a Research Paper
  • Average Stipend for Research/Academic Internships
  • These Institutes Offer Remote Research/Academic Internships
  • All Blog Posts
  • Research Career
  • Research Conference
  • Research Internship
  • Research Journal
  • Research Tools
  • Uncategorized
  • Research Conferences
  • Research Journals
  • Research Grants
  • Internships
  • Research Internships
  • Email Templates
  • Conferences
  • Blog Partners
  • Privacy Policy

Copyright © 2024 Research Voyage

Design by ThemesDNA.com

close-link

Logo

Tips for writing a PhD dissertation: FAQs answered

From how to choose a topic to writing the abstract and managing work-life balance through the years it takes to complete a doctorate, here we collect expert advice to get you through the PhD writing process

Campus team's avatar

Campus team

Additional links.

  • More on this topic

Figures with light bulb

You may also like

Signposts for help, support, advice and guidance

Popular resources

.css-1txxx8u{overflow:hidden;max-height:81px;text-indent:0px;} Change is coming, whether higher education likes it or not

Ai and assessment redesign: a four-step process, upgrade your teaching: four instant improvement tips, the cruel optimism of research careers: how to support contract workers, create an onboarding programme for neurodivergent students.

Embarking on a PhD is “probably the most challenging task that a young scholar attempts to do”, write Mark Stephan Felix and Ian Smith in their practical guide to dissertation and thesis writing. After years of reading and research to answer a specific question or proposition, the candidate will submit about 80,000 words that explain their methods and results and demonstrate their unique contribution to knowledge. Here are the answers to frequently asked questions about writing a doctoral thesis or dissertation.

What’s the difference between a dissertation and a thesis?

Whatever the genre of the doctorate, a PhD must offer an original contribution to knowledge. The terms “dissertation” and “thesis” both refer to the long-form piece of work produced at the end of a research project and are often used interchangeably. Which one is used might depend on the country, discipline or university. In the UK, “thesis” is generally used for the work done for a PhD, while a “dissertation” is written for a master’s degree. The US did the same until the 1960s, says Oxbridge Essays, when the convention switched, and references appeared to a “master’s thesis” and “doctoral dissertation”. To complicate matters further, undergraduate long essays are also sometimes referred to as a thesis or dissertation.

The Oxford English Dictionary defines “thesis” as “a dissertation, especially by a candidate for a degree” and “dissertation” as “a detailed discourse on a subject, especially one submitted in partial fulfilment of the requirements of a degree or diploma”.

  • Ten platinum rules for PhD supervisors
  • Fostering freedom in PhD students: how supervisors can shape accessible paths for doctoral research
  • Lessons from students on effective research supervision

The title “doctor of philosophy”, incidentally, comes from the degree’s origins, write Dr Felix, an associate professor at Mahidol University in Thailand, and Dr Smith, retired associate professor of education at the University of Sydney , whose co-authored guide focuses on the social sciences. The PhD was first awarded in the 19th century by the philosophy departments of German universities, which at that time taught science, social science and liberal arts.

How long should a PhD thesis be?

A PhD thesis (or dissertation) is typically 60,000 to 120,000 words ( 100 to 300 pages in length ) organised into chapters, divisions and subdivisions (with roughly 10,000 words per chapter) – from introduction (with clear aims and objectives) to conclusion.

The structure of a dissertation will vary depending on discipline (humanities, social sciences and STEM all have their own conventions), location and institution. Examples and guides to structure proliferate online. The University of Salford , for example, lists: title page, declaration, acknowledgements, abstract, table of contents, lists of figures, tables and abbreviations (where needed), chapters, appendices and references.

A scientific-style thesis will likely need: introduction, literature review, materials and methods, results, discussion, bibliography and references.

As well as checking the overall criteria and expectations of your institution for your research, consult your school handbook for the required length and format (font, layout conventions and so on) for your dissertation.

A PhD takes three to four years to complete; this might extend to six to eight years for a part-time doctorate.

What are the steps for completing a PhD?

Before you get started in earnest , you’ll likely have found a potential supervisor, who will guide your PhD journey, and done a research proposal (which outlines what you plan to research and how) as part of your application, as well as a literature review of existing scholarship in the field, which may form part of your final submission.

In the UK, PhD candidates undertake original research and write the results in a thesis or dissertation, says author and vlogger Simon Clark , who posted videos to YouTube throughout his own PhD journey . Then they submit the thesis in hard copy and attend the viva voce (which is Latin for “living voice” and is also called an oral defence or doctoral defence) to convince the examiners that their work is original, understood and all their own. Afterwards, if necessary, they make changes and resubmit. If the changes are approved, the degree is awarded.

The steps are similar in Australia , although candidates are mostly assessed on their thesis only; some universities may include taught courses, and some use a viva voce. A PhD in Australia usually takes three years full time.

In the US, the PhD process begins with taught classes (similar to a taught master’s) and a comprehensive exam (called a “field exam” or “dissertation qualifying exam”) before the candidate embarks on their original research. The whole journey takes four to six years.

A PhD candidate will need three skills and attitudes to get through their doctoral studies, says Tara Brabazon , professor of cultural studies at Flinders University in Australia who has written extensively about the PhD journey :

  • master the academic foundational skills (research, writing, ability to navigate different modalities)
  • time-management skills and the ability to focus on reading and writing
  • determined motivation to do a PhD.

Socrates' methods can still help university student in the battle with misinformation

How do I choose the topic for my PhD dissertation or thesis?

It’s important to find a topic that will sustain your interest for the years it will take to complete a PhD. “Finding a sustainable topic is the most important thing you [as a PhD student] would do,” says Dr Brabazon in a video for Times Higher Education . “Write down on a big piece of paper all the topics, all the ideas, all the questions that really interest you, and start to cross out all the ones that might just be a passing interest.” Also, she says, impose the “Who cares? Who gives a damn?” question to decide if the topic will be useful in a future academic career.

The availability of funding and scholarships is also often an important factor in this decision, says veteran PhD supervisor Richard Godwin, from Harper Adams University .

Define a gap in knowledge – and one that can be questioned, explored, researched and written about in the time available to you, says Gina Wisker, head of the Centre for Learning and Teaching at the University of Brighton. “Set some boundaries,” she advises. “Don’t try to ask everything related to your topic in every way.”

James Hartley, research professor in psychology at Keele University, says it can also be useful to think about topics that spark general interest. If you do pick something that taps into the zeitgeist, your findings are more likely to be noticed.

You also need to find someone else who is interested in it, too. For STEM candidates , this will probably be a case of joining a team of people working in a similar area where, ideally, scholarship funding is available. A centre for doctoral training (CDT) or doctoral training partnership (DTP) will advertise research projects. For those in the liberal arts and social sciences, it will be a matter of identifying a suitable supervisor .

Avoid topics that are too broad (hunger across a whole country, for example) or too narrow (hunger in a single street) to yield useful solutions of academic significance, write Mark Stephan Felix and Ian Smith. And ensure that you’re not repeating previous research or trying to solve a problem that has already been answered. A PhD thesis must be original.

What is a thesis proposal?

After you have read widely to refine your topic and ensure that it and your research methods are original, and discussed your project with a (potential) supervisor, you’re ready to write a thesis proposal , a document of 1,500 to 3,000 words that sets out the proposed direction of your research. In the UK, a research proposal is usually part of the application process for admission to a research degree. As with the final dissertation itself, format varies among disciplines, institutions and countries but will usually contain title page, aims, literature review, methodology, timetable and bibliography. Examples of research proposals are available online.

How to write an abstract for a dissertation or thesis

The abstract presents your thesis to the wider world – and as such may be its most important element , says the NUI Galway writing guide. It outlines the why, how, what and so what of the thesis . Unlike the introduction, which provides background but not research findings, the abstract summarises all sections of the dissertation in a concise, thorough, focused way and demonstrates how well the writer understands their material. Check word-length limits with your university – and stick to them. About 300 to 500 words is a rough guide ­– but it can be up to 1,000 words.

The abstract is also important for selection and indexing of your thesis, according to the University of Melbourne guide , so be sure to include searchable keywords.

It is the first thing to be read but the last element you should write. However, Pat Thomson , professor of education at the University of Nottingham , advises that it is not something to be tackled at the last minute.

How to write a stellar conclusion

As well as chapter conclusions, a thesis often has an overall conclusion to draw together the key points covered and to reflect on the unique contribution to knowledge. It can comment on future implications of the research and open up new ideas emanating from the work. It is shorter and more general than the discussion chapter , says online editing site Scribbr, and reiterates how the work answers the main question posed at the beginning of the thesis. The conclusion chapter also often discusses the limitations of the research (time, scope, word limit, access) in a constructive manner.

It can be useful to keep a collection of ideas as you go – in the online forum DoctoralWriting SIG , academic developer Claire Aitchison, of the University of South Australia , suggests using a “conclusions bank” for themes and inspirations, and using free-writing to keep this final section fresh. (Just when you feel you’ve run out of steam.) Avoid aggrandising or exaggerating the impact of your work. It should remind the reader what has been done, and why it matters.

How to format a bibliography (or where to find a reliable model)

Most universities use a preferred style of references , writes THE associate editor Ingrid Curl. Make sure you know what this is and follow it. “One of the most common errors in academic writing is to cite papers in the text that do not then appear in the bibliography. All references in your thesis need to be cross-checked with the bibliography before submission. Using a database during your research can save a great deal of time in the writing-up process.”

A bibliography contains not only works cited explicitly but also those that have informed or contributed to the research – and as such illustrates its scope; works are not limited to written publications but include sources such as film or visual art.

Examiners can start marking from the back of the script, writes Dr Brabazon. “Just as cooks are judged by their ingredients and implements, we judge doctoral students by the calibre of their sources,” she advises. She also says that candidates should be prepared to speak in an oral examination of the PhD about any texts included in their bibliography, especially if there is a disconnect between the thesis and the texts listed.

Can I use informal language in my PhD?

Don’t write like a stereotypical academic , say Kevin Haggerty, professor of sociology at the University of Alberta , and Aaron Doyle, associate professor in sociology at Carleton University , in their tongue-in-cheek guide to the PhD journey. “If you cannot write clearly and persuasively, everything about PhD study becomes harder.” Avoid jargon, exotic words, passive voice and long, convoluted sentences – and work on it consistently. “Writing is like playing guitar; it can improve only through consistent, concerted effort.”

Be deliberate and take care with your writing . “Write your first draft, leave it and then come back to it with a critical eye. Look objectively at the writing and read it closely for style and sense,” advises THE ’s Ms Curl. “Look out for common errors such as dangling modifiers, subject-verb disagreement and inconsistency. If you are too involved with the text to be able to take a step back and do this, then ask a friend or colleague to read it with a critical eye. Remember Hemingway’s advice: ‘Prose is architecture, not interior decoration.’ Clarity is key.”

How often should a PhD candidate meet with their supervisor?

Since the PhD supervisor provides a range of support and advice – including on research techniques, planning and submission – regular formal supervisions are essential, as is establishing a line of contact such as email if the candidate needs help or advice outside arranged times. The frequency varies according to university, discipline and individual scholars.

Once a week is ideal, says Dr Brabazon. She also advocates a two-hour initial meeting to establish the foundations of the candidate-supervisor relationship .

The University of Edinburgh guide to writing a thesis suggests that creating a timetable of supervisor meetings right at the beginning of the research process will allow candidates to ensure that their work stays on track throughout. The meetings are also the place to get regular feedback on draft chapters.

“A clear structure and a solid framework are vital for research,” writes Dr Godwin on THE Campus . Use your supervisor to establish this and provide a realistic view of what can be achieved. “It is vital to help students identify the true scientific merit, the practical significance of their work and its value to society.”

How to proofread your dissertation (what to look for)

Proofreading is the final step before printing and submission. Give yourself time to ensure that your work is the best it can be . Don’t leave proofreading to the last minute; ideally, break it up into a few close-reading sessions. Find a quiet place without distractions. A checklist can help ensure that all aspects are covered.

Proofing is often helped by a change of format – so it can be easier to read a printout rather than working off the screen – or by reading sections out of order. Fresh eyes are better at spotting typographical errors and inconsistencies, so leave time between writing and proofreading. Check with your university’s policies before asking another person to proofread your thesis for you.

As well as close details such as spelling and grammar, check that all sections are complete, all required elements are included , and nothing is repeated or redundant. Don’t forget to check headings and subheadings. Does the text flow from one section to another? Is the structure clear? Is the work a coherent whole with a clear line throughout?

Ensure consistency in, for example, UK v US spellings, capitalisation, format, numbers (digits or words, commas, units of measurement), contractions, italics and hyphenation. Spellchecks and online plagiarism checkers are also your friend.

Image of students being support to cross a gap in their learning

How do you manage your time to complete a PhD dissertation?

Treat your PhD like a full-time job, that is, with an eight-hour working day. Within that, you’ll need to plan your time in a way that gives a sense of progress . Setbacks and periods where it feels as if you are treading water are all but inevitable, so keeping track of small wins is important, writes A Happy PhD blogger Luis P. Prieto.

Be specific with your goals – use the SMART acronym (specific, measurable, attainable, relevant and timely).

And it’s never too soon to start writing – even if early drafts are overwritten and discarded.

“ Write little and write often . Many of us make the mistake of taking to writing as one would take to a sprint, in other words, with relatively short bursts of intense activity. Whilst this can prove productive, generally speaking it is not sustainable…In addition to sustaining your activity, writing little bits on a frequent basis ensures that you progress with your thinking. The comfort of remaining in abstract thought is common; writing forces us to concretise our thinking,” says Christian Gilliam, AHSS researcher developer at the University of Cambridge ’s Centre for Teaching and Learning.

Make time to write. “If you are more alert early in the day, find times that suit you in the morning; if you are a ‘night person’, block out some writing sessions in the evenings,” advises NUI Galway’s Dermot Burns, a lecturer in English and creative arts. Set targets, keep daily notes of experiment details that you will need in your thesis, don’t confuse writing with editing or revising – and always back up your work.

What work-life balance tips should I follow to complete my dissertation?

During your PhD programme, you may have opportunities to take part in professional development activities, such as teaching, attending academic conferences and publishing your work. Your research may include residencies, field trips or archive visits. This will require time-management skills as well as prioritising where you devote your energy and factoring in rest and relaxation. Organise your routine to suit your needs , and plan for steady and regular progress.

How to deal with setbacks while writing a thesis or dissertation

Have a contingency plan for delays or roadblocks such as unexpected results.

Accept that writing is messy, first drafts are imperfect, and writer’s block is inevitable, says Dr Burns. His tips for breaking it include relaxation to free your mind from clutter, writing a plan and drawing a mind map of key points for clarity. He also advises feedback, reflection and revision: “Progressing from a rough version of your thoughts to a superior and workable text takes time, effort, different perspectives and some expertise.”

“Academia can be a relentlessly brutal merry-go-round of rejection, rebuttal and failure,” writes Lorraine Hope , professor of applied cognitive psychology at the University of Portsmouth, on THE Campus. Resilience is important. Ensure that you and your supervisor have a relationship that supports open, frank, judgement-free communication.

If you would like advice and insight from academics and university staff delivered direct to your inbox each week, sign up for the Campus newsletter .

Authoring a PhD Thesis: How to Plan, Draft, Write and Finish a Doctoral Dissertation (2003), by Patrick Dunleavy

Writing Your Dissertation in Fifteen Minutes a Day: A Guide to Starting, Revising, and Finishing Your Doctoral Thesis (1998), by Joan Balker

Challenges in Writing Your Dissertation: Coping with the Emotional, Interpersonal, and Spiritual Struggles (2015), by Noelle Sterne

Change is coming, whether higher education likes it or not

The iscanner app supports the academic community in information sharing and management, support students with caring responsibilities in he, step up to support students with disabilities, emotions and learning: what role do emotions play in how and why students learn, the secret to timely, relevant, inclusive communication with students.

Register for free

and unlock a host of features on the THE site

Department of Biology

PhD and MPhil in Biology

  Length Start dates ( )
PhD

3-4 years full-time
6-8 years part-time

January
April
July
September

MPhil

2 years full-time
4 years part-time

January
April
July
September

Biological research has a critical role to play in securing a sustainable future. Our interdisciplinary research contributes to solutions for three major global challenges - health and disease, sustainable food and fuel, and environmental change. 

Every step of our research is carried out with the help of our postgraduate students. No matter which area of biology you specialise in, you will be working alongside some of the world’s biggest names in their respective fields, at the forefront of scientific exploration. 

Your research

A PhD thesis can be up to 80,000 words long and must contain a substantial original contribution to scientific knowledge or understanding.

An MPhil thesis can be up to 60,000 words and is expected to display a good general knowledge of the field of study; a comprehensive knowledge of some part or aspect of the field of study; and a recognisable original contribution to knowledge or understanding.

Both theses are assessed by internal and external examiners, who will feed back on your work, and may require corrections. You will sit a final oral examination, known as a viva, within three months of submitting your thesis.

[email protected] +44 (0)1904 328546

Related links

  • Research degree funding
  • Accommodation
  • International students
  • Life at York
  • How to apply

Biological Sciences is ranked in the top ten overall in the UK

in the Times Higher Education's ranking of the latest Research Excellence Framework (REF) results 2021.

Access to state-of-the-art facilities

and expert advice through our Bioscience Technology Facility.

Dedicated study spaces

including a flexible workshop and creative learning lounge.

Explore funding for postgraduate researchers in Biology and wider postgraduate support.

Supervision

You'll be assigned a supervisor who will guide your work throughout your time with us. We'll match your research interests to their area of expertise.

Training and support

Throughout your course you'll have a supervisor to support you and guide your research. 

Work on your thesis is monitored by regular meetings of the Thesis Advisory Panel (at least once every six months for a full-time student and once a year for part-time students). The panel will review the progress of your research and provide additional advice and guidance.

Our research degrees include a cohort-based training programme to provide you with project-specific and transferable skills for your future career. This includes reading and appraising scientific literature, presenting confidently to a scientific audience, and writing accounts of your research.

biology phd thesis length

Course location

This course is run by the Department of Biology.

The Department of Biology is located on Campus West .

Entry requirements

To apply for these courses you should hold, or expect to hold, an honours degree in any biological subject with a 2:1 or first-class honours (or overseas equivalent).

We will consider applicants with backgrounds in chemistry, computer science, electronics, environment, mathematics, medicine or physics who can demonstrate a commitment to the biosciences.

English language requirements

If English is not your first language you must provide evidence of your ability.

Check your English language requirements

Apply for the PhD 

Apply for the MPhil  

Take a look at the supporting documents you may need for your application.

Find out more about how to apply .

Identify a supervisor

Before applying, you need to identify a research project that interests you. Look at the lists of academic staff in each of our research groups to see if there is a prospective supervisor working in an area which relates to your own research interests. You're welcome to email a prospective supervisor informally to find out more about their research or to ask if they might be considering taking on a postgraduate researcher.

Find a supervisor

As part of your application, you'll be invited to an interview (either face-to-face or online). As part of the interview process, you will usually be expected to make a short presentation on a recent piece of research you have conducted. If your first language is not English, you should be fluent enough to speak confidently about your intended area of research and discuss your background and interests with a panel of academics.

Careers and skills

Our dedicated careers team offer specific support including a programme of professional researcher development and careers workshops and 1:1 career support sessions. They will help you to build up your employability portfolio and to engage in activities that will build up your skills and experience within and outside of your research work.

You will have the opportunity to gain experience of demonstrating to undergraduate and taught Masters students, and receive practical advice on career planning. We hold a series of research seminars across all disciplines which are open to all researchers. There are also opportunities to present your own work through posters and talks.

A research degree is essential if you plan a career as an independent researcher with responsibility for your own research programme, whether in academia, research institutes or industry.

Career opportunities

Our graduates have gone on to work in a wide range of sectors, including:

  • clinical science
  • biotechnology
  • scientific research
  • postdoctoral positions
  • teaching and lecturing.

Discover York

biology phd thesis length

We offer a range of campus accommodation to suit you and your budget, from economy to deluxe.

biology phd thesis length

Discover more about our researchers, facilities and why York is the perfect choice for your research degree.

biology phd thesis length

Graduate Research School

Connect with researchers across all disciplines to get the most out of your research project.

Meet us online or on campus

Find out all you need to know about applying to York

Scholarships

Find scholarships to support your studies

Information and Forms

There are two tracks within the Biology Ph.D. program:

  • Cell, Molecular and Organismal Biology
  • Ecology and Evolution

(Previously a part of the Department of Biology Hopkins Marine Station is now a part of the Oceans Department within  Stanford Doerr School of Sustainability )

Some requirements are the same across concentrations, and other requirements are concentration-specific. Please review the Ph.D. Handbook for specific details.

General Information and Forms

  • Graduate Academic Policies and Procedures
  • 2023-2024 Biology PhD Handbook
  • CMOB Elective Course List 
  • Registrar's Office Forms  (SUNet ID login required)

Teaching Assistant Evaluations (All years)

  • TA Pre-Course Meeting Form  - Due within the first two weeks of starting each TA position
  • TA Evaluation Form  - Due after each TA position

Special Circumstance Forms

  • Graduation Extension Petition  (if applicable)
  • Change of Advisor or Reading Committee Member Form  (if applicable)

First Year Forms

Forms for all first year students.

  • Lab Rotation Evaulation  - Due after each rotation, if applicable
  • NSF Application  - If eligible, Due October 18 (Life Sciences) or October 22 (Geosciences)
  • Dissertation Lab Decision - Due May 1
  • First Year Evaluation and Planning Form  - Due May 1
  • IDP Meeting  - Due August 1

Additional Forms for EcoEvo & Hopkins Students

  • First Year Advising Committee Form  - Due within the first 2 weeks of Autumn, Winter, and Spring quarters
  • Seminar/Talk Evaluation Form  - Due June 1
  • First Year Paper Evaluation Form  - Due June 1

Second Year Forms

In second year you will complete the qualification exam (forms below) then advance to candidacy (forms at bottom),  forms for cmob students.

  • 2nd Year Qualifying Exam Evaluation Form

 Forms for Eco/Evo Students

  • Dissertation Proposal Paper - Due May 15 (electronic Submission)
  • 2nd Year Qualifying Exam Evaluation Form -

 Forms for Hopkins Students

Forms for all second year students.

  • Application for Candidacy  
  • Dissertation Reading Committee Form  
  • Non-AC Committee Member Form  (if applicable)

Third Year Forms

Forms for all third year students.

  • Committee Meeting Form  (annual)

Additional Forms for CMOB Students

  • Seminar Evaluation Form  

Fourth Year Forms

Forms for all fourth year students.

  • Request for TGR Status Form  - Due March 15 (after completion of 135 units and all of the first through third year requirements)
  • Committee Meeting Form (Annual)

Fifth & Sixth Year Forms

Forms for all fifth and sixth year students.

  • Committee Meeting Form  (Semi-annual)
  • Committee Meeting Form  (Semi-annual)
  • IDP Meeting  - August 1

Dissertation Defense & Submission

Dissertation deadlines.

  • Draft of Dissertation - Send to committee 30 days before defense
  • University Oral Exam Form found here - Due 2 weeks before defense
  • Directions for Preparing and Submitting the Dissertation  -  Check due dates for each quarter

Archived Ph.D. Handbooks:

  • .   2020-21
  • 2018-19 

x == (s || z). You say it kwontized

Rule of Three: advice on writing a PhD thesis

PhD students sometimes get the same bad advice on writing their thesis. I call this advice the Rule of Three . Typically, they get told that their thesis:

  • Will take 3 months to write
  • Should have 3 results chapters
  • Should be 300 pages

These bits of advice have one thing in common: they are all wrong.

  • If you have been organised (see below), it should not take 3 months to write a PhD thesis. It certainly shouldn’t involve leaving the lab 3 months before your hand-in date to write up.
  • Theses can have one results chapter or they can have more. How many chapters depends on your project, and your results. Trying to make three results chapters out of one chapter ends up in a weak or overlong thesis.
  • A thesis is like a piece of string and it will be as long as it needs to be. Aim for brevity and not producing a magnus opus (see below).
Disclaimer: what follows is some different advice. As with all “advice”, your mileage will vary. It is written for the people in my lab but likely applies to UK PhD students doing biomedical research.

Rule 1: aim for a thesis that is good enough

Who will read your thesis? Two people. Your examiners. OK, some parts – such as the Methods section – will be useful to future lab members (although with electronic lab notebooks this function is becoming redundant). Maybe your thesis will be downloaded by someone from the repository, but essentially, it will only be read by your examiners.

How long does an examiner spend reading your thesis? A few hours. One day maximum. They simply have no more time. Do you really want to spend three months of your life writing something that will be read for just a few hours by two people?

It’s for these reasons that spending too much effort writing a perfect thesis is a waste of time. It just needs to be good enough.

As well as being just good enough, it only needs to be long enough . A big mistake students make is to produce a really long thesis because they think that that is what theses should be (rule of 3). What happens is the examiner will receive the thesis, look at how many pages there are, subtract the bibliography, and their heart will sink if it is too long.

You might now be wondering: is writing a thesis a waste of time?

No, because you have to do it to get your PhD.

No, because you learn important writing skills. You also learn how to assemble a large document (it’s often how students learn to use Word properly or up their LaTeX game). It’s good training for writing papers and other technical documents down the line. Employers know this when they hire you.

But that is about it. So you just need to write something that is good enough to pass.

Rule 2: prioritise papers and the thesis will follow

Papers are the priority. They are more useful to you and to your PI. But this advice isn’t motivated by self-interest. If you go into the viva and the work in your thesis is already peer reviewed and published, it’s harder for the examiners to criticise it. At least, they will not approach your thesis with the question: is this work publishable? This is one criteria for passing your PhD, so demonstrating that it is publishable means you are (almost) there.

This was the one bit of advice I received when doing my PhD and it is still true today. OK, it is harder these days to get a first author paper published before you submit your thesis. However a preprint on bioRxiv before you begin writing will help you to prepare your thesis and will still tick the publishable box.

How long should it take to write your thesis?

There is tension here because you are at your most useful in the lab as you near the end of your PhD. One week of labwork now is worth one month (or more) earlier in your PhD. You are most valuable to your lab/PI/science/career at this point and keeping working in the lab will yield more rewards. But it won’t get your thesis written.

The first bit of writing is busywork and can be done around lab work. “Deep writing” and reading does need time away for most students.

If you have only collected data in the lab and not analysed it, if you’ve not presented your work very often, if you are disorganised… yes, it will take you a full three months to write your thesis.

All the folks in my lab are encouraged to get figures ready, analyse as they go and they also give regular talks. It should not take anyone in this position three months away from the lab to write their thesis.

Agreeing a timeline with your PI for when you begin writing is really important. Regular deadlines and a commitment to timely feedback from your supervisor make thesis writing easier. The discussion needs to be based on facts though. Often students want to budget a lot of time to writing, because of the rule of 3 or because they believe they are “bad at writing”. It helps to see some evidence. Writing draft chapters earlier in the PhD – which is a requirement at some universities – can reveal difficulties and weaknesses.

Reality check

If you hear the rule of 3 from everyone and your supervisor is giving you different advice. It might be time for a reality check. Have a look at past theses from the lab. How long were they? How many chapters? Information is good.

YearResults ChaptersPages excl. bibliography/appendicesPapers associated with thesis
201132081 published paper. Other work published later.
201322031 published paper. Appendix contains a further paper and methods paper.
201631331 published, 1 in preparation.
20163941 paper in revision.
201721381 paper submitted. Contributed to three other papers
201831212 papers (but not published or submitted at the time). Appendix with one paper they contributed to.

You can see that all theses are fewer than 300 pages in length, many substantially so. Four have three chapters and two have two. Although looking closer, two of the theses with three chapters use a results chapter as an expanded methods chapter.

Ultimately, the thesis is your work but you will get input from your supervisor. Regardless of what is written here or how many people tell you about the rule of 3, your supervisor will have their own ideas about how your thesis should be. Agreeing a sensible plan with them is the way to get started productively.

Getting started

This is not a comprehensive guide but in order to write a good enough thesis, you first need a plan.

  • Make a figure list. This should be every single figure you can think of. You can cross off ones you don’t need later if they don’t fit or are insignificant.
  • Plan the narrative. There is usually more than one way to put together the figures to make a thesis. Be prepared for this to change after you start writing! Sometimes the writing process reveals ways in which the narrative should be rearranged.
  • How many results chapters? Start with the idea that you will have one. Does it need dividing? If yes, then what are the titles of the two chapters? If you have difficulty titling them you may need to split to a 3rd.

Now you have a plan. It’s time to get going.

Set some goals – but make them small. Having a goal of “I am going to complete my thesis” is too demoralising. You need to feel like you are making progress constantly to stay motivated. Break it down into smaller chunks. “I will finish this chapter by next Friday”. “I will write the cloning section this morning and then go for a walk”.

Write the materials and methods first . It’s the easiest bit to write because it is all technical writing with little wordcraft required. You can fit it around labwork. In fact, it is easier to write whilst in the lab because you can look up all the stuff you need. Importantly, it gets over the “blank page syndrome”.

Next get your figures together . This should already be done if you have been organised.

Then write the figure legends . You already have the title for each figure from your plan. All you need to do is describe each panel. Again, quite low energy writing required for this task.

Now write the results sections ! This is the same way that we put papers together. The results parts of the thesis are more extended but in principle you will guide the reader though the figures that you’ve made. Remember, you already have the legends written. So you are already partly on your way.

Time to regroup . At this point you can refine your plan for the introduction and check the rest of your plan still makes sense. Now is the time for some deep writing and reading.

The post title comes from “Rule of Three” by The Lemonheads.

biology phd thesis length

Thesis/dissertation committees

  • Graduate resources overview

Students are guided by faculty mentors who advise them through their degree program, called supervisory committees. Students should identify their committees early in their graduate career and look to them for guidance in their graduate work. Once students have selected a supervisory committee, they must add each member in their iPOS, in the faculty committee section, and submit for approval. Approval then goes through levels of review as needed within SOLS and the Graduate College.

Your initial iPOS submission only requires your chair or co-chairs be added. You will later need to add the rest of your committee members by the deadlines described for your specific program in the handbook. These deadlines are summarized below. If you need an extension or have questions about these deadlines, please contact your faculty advisor and program director(s).

PhD dissertation committee submissions

  • Animal Behavior PhD: End of 2nd semester
  • Biology PhD: End of 2nd semester
  • Biology and Society PhD: End of 4th semester and at least 1 semester prior to planned comprehensive exams and dissertation prospectus
  • Environmental Life Sciences PhD: End of 2nd semester
  • Evolutionary Biology PhD: End of 2nd semester
  • History and Philosophy of Science PhD: End of 4th semester and at least 1 semester prior to planned comprehensive exams and dissertation prospectus
  • Microbiology PhD: End of 3rd semester
  • Molecular and Cellular Biology PhD: End of 2nd semester
  • Neuroscience PhD: End of 2nd semester  

MS thesis committee submissions

  • Biology MS (Research & Thesis): End of 1st semester
  • Biology and Society MS: End of 2nd semester
  • Microbiology MS: End of 1st semester
  • Molecular and Cellular Biology MS: End of 1st semester
  • Plant Biology and Conservation MS: End of 1st semester  

Committee chair

For most students, their thesis or dissertation committee Chair is their faculty/research advisor. This is typically agreed upon during the admissions process, though some students complete rotations or coursework first. The faculty/research advisor must be a member of the ASU Graduate Faculty and endorsed to chair or co-chair a committee in the student’s degree program. You can see this by searching their name in the iPOS > Faculty Committee tab. If they are endorsed to chair, no co-chair is needed. If they endorsed to co-chair only (which is typically the case for advisors at partner institutions like Mayo, BNI, Desert Botanical Garden, etc.), then a tenure-track SOLS faculty member must be chosen to serve as the other co-chair.  If your planned committee chair or co-chair does not have the proper endorsement or is not pulling up when you search them in your iPOS, the proper endorsement may just need to be added. This is commonly the case for newer faculty or external research advisors. In some cases, this could be because they are not eligible for the role you are attempting to add them for. If this is the case for you, please fill out the request form linked here in order to initiate the process of properly endorsing your committee chair or co-chair.  

PhD committees

  • *Animal Behavior PhD is the only exception to this, as it required at least 5 total members
  • 3 of the 4 (or 5 for Animal Behavior) individuals on the committee should be ASU tenure-track faculty
  • 2 of the 4 (or 5 for Animal Behavior) individuals on the committee should be ASU tenure-track and program-specific faculty (i.e., if you are a Biology PhD student, 2 of the 4 folks on your committee must be tenure-track faculty in the biology program)
  • In some cases, an external committee member may be approved as the 4th (or 5th for Animal Behavior) committee member with justification from the student and approval from the Program Director, Graduate Associate Director, and Graduate College
  • While you may have more than 4 (or 5 for Animal Behavior) committee members, please consider your justification for doing so, as coordinating schedules for large committees can place undue burden on students

Certain programs have program-specific rules with their committee expectations. Please review your program's requirements in the SOLS Handbook to ensure you meet requirements. If you have a special request, please consult with your Program Director(s).  

MS thesis committees

  • Must have at least 3 total members (1 committee chair and 2 members, or 2 committee co-chairs and 1 member)
  • 2 of the 3 individuals on the committee should be ASU tenure-track, program-specific faculty (i.e., if you are a Biology MS student, 2 of the 3 folks on your committee must be tenure-track faculty in the biology program)
  • In some cases, an external committee member may be approved as the 3rd committee member with justification from the student and approval from the Program Director, Graduate Associate Director, and Graduate College
  • While you may have more than 3 committee members, please consider your justification for doing so, as coordinating schedules for large committees can place undue burden on students Certain programs have program-specific rules with their committee expectations. Please review your program's requirements in the SOLS Handbook to ensure you meet requirements. If you have a special request, please consult with your Program Director(s).

MS Biology Capstone committees

If you are enrolled in the MS Biology - Coursework & Capstone track, you are not required to have a formal 3-person supervisory committee. Instead, you are required to have 1 committee Chair. Typically, this is the current Program Director for the MS Biology - Coursework & Capstone program: Dr. Stephen Pratt. However, if you are working on a capstone project under the guidance of another SOLS faculty member, it may be appropriate to add them as your committee Chair. Please contact your academic advisor or [email protected] if you would like guidance on this matter.

Adding new or external committee member(s)

If you have an external individual you would like to add to your committee or someone internal that does not have approval yet, please fill our the faculty committee request form by clicking the button below. In order to submit your request, you will need an up to date CV or resume of the individual you are requesting approval for, their date of birth if they are not ASU personnel, or their ASU ID number if they are ASU personnel. Once you submit your request via this form, the SOLS Graduate Office will route it through the necessary channels for approval including Program Directors, the Graduate Associate Director, and the Graduate College. You'll be notified once approved so you can then update your iPOS accordingly.  

UNSW Logo

Doctor of Philosophy

Students studying at the library

The doctoral degree programs within the School of Biotechnology & Biomolecular Sciences provide the highest level of training in key areas of scientific research. 

Program objectives

Students work on an independent research project encompassing the broader interests of one of the research teams within the school. In the early stages of the program, students receive close supervision and guidance in the management of their project. In the later stages, however, students are encouraged to exercise initiative and demonstrate originality. In the last year of the program, the candidate should be able to work independently and be guided rather than directed by the supervisor.

Entrance requirements

The minimum entry requirement for a PhD is a four-year Bachelor’s degree with First or Upper Second Class honours or completion of a Bachelor degree and substantial laboratory experience.

Applicants must seek and obtain agreement of a BABS academic to be their supervisor prior to applying; this is a key requirement to gain acceptance. View the  BABS HDR Information Booklet  for supervisors and research areas. Identifying and negotiating with prospective supervisors is key to gaining admission, and you will need to align your topic of interest with the research area of one of our School’s academics. 

Applicants must also provide evidence that their English language ability meets the minimum requirements for admission: English language requirements .

Further information can be obtained from the Graduate Research School (GRS) .

PhD degrees

  • PhD – Biochemistry & Molecular Genetics (1410)
  • PhD – Bioinformatics (1683)
  • PhD – Biotechnology (1036)
  • PhD – Microbiology & Immunology (1440)

Program requirements

A Doctor of Philosophy Degree requires three years full-time study and completion of a written thesis. The length of a doctoral thesis is normally around 100,000 words. The thesis is reviewed by members of the Australian and international scientific academic community. In the course of their research, PhD students must make a distinct contribution to the knowledge within their specific discipline. Ideally, this will result in the publication of original research findings in peer-reviewed journals of international standing.

Domestic students: The Australian Government's Research Training Scheme (RTS) provides Commonwealth-funded higher degree by research (HDR) candidates with an exemption from tuition fees for the duration of their PhD by research for a maximum period of four years' full-time-equivalent study. Domestic students are liable for the UNSW Student Services Amenities Fee .

International students: Full details can be found on the GRS Fees and Costs page for International Postgraduate Research Candidates. Note that international students are exempt from the UNSW Student Services Amenities Fee.

How to apply

Applicants may view the application procedures on the GRS website .

View the application deadlines.  

PhD stories

The School of Biotechnology & Biomolecular Sciences has been home to groundbreaking research for more than 50 years. The hundreds of honours, masters and PhD theses presented below hold this legacy.

Student reading book at the library

Bodleian Libraries

  • Bodleian Libraries
  • Oxford LibGuides
  • Theses and Dissertations

Biology: Theses and Dissertations

  • Journals and Conference Proceedings
  • Free Online Resources
  • Biology Archives

Introduction

Theses and dissertations are documents that present an author's research findings, which are submitted to the University in support of their academic degree. They are very useful to consult when carrying out your own research because they:

  • provide a springboard to scope existing literature
  • provide inspiration for the finished product
  • show you the evolution of an author's ideas over time
  • provide relevant and up-to-date research (for recent theses and dissertations)

On this page you will find guidance on how to search for and access theses and dissertations in the Bodleian Libraries and beyond.

Definitions

Terms you may encounter in your research.

Thesis: In the UK, a thesis is normally a document that presents an author's research findings as part of a doctoral or research programme.

Dissertation: In the UK, a dissertation is normally a document that presents an author's research findings as part of an undergraduate or master's programme.

DPhil: An abbreviation for Doctor of Philosophy, which is an advanced research qualification. You may also see it referred to as PhD.

ORA: The Oxford University Research Archive , an institutional repository for the University of Oxford's research output including digital theses.

Theses and dissertations

  • Reading theses and dissertations in the Bodleian Libraries

The Bodleian Libraries collection holds DPhil, MLitt and MPhil theses deposited at the University of Oxford, which you can consult. You may also be interested to read theses and dissertations beyond the University of Oxford, some of which can be read online, or you can request an inter-library loan.

Help with theses and dissertations

To find out more about how to find and access theses and dissertations in the Bodleian Libraries and beyond, we recommend the following:

  • Bodleian Libraries theses and dissertations Links to information on accessing the Bodleian Libraries collections of Oxford, UK, US and other international theses.
  • Oxford University Research Archive guide An introduction to ORA, the Oxford University Research Archive. ORA is the institutional repository for the University of Oxford and is home to the scholarly research output of its members. ORA also is the home of Oxford digital theses.
  • Help & guidance for digital theses Information on copyright, how to deposit your thesis in ORA and other important matters
  • Guide to copyright The Bodleian Libraries' Quick guide to copyright and digital sources.

Depositing your thesis

It is mandatory for students completing a research degree at the University of Oxford (registered to a programme of study on or after 1st October 2007) to deposit an electronic copy of their theses with the Oxford University Research Archive (ORA) in order to meet the requirements of their award. To find out more, visit the Oxford University Research Archive guide.

  • Oxford University Research Archive guide
  • << Previous: Free Online Resources
  • Next: Biology Archives >>
  • Last Updated: Aug 23, 2024 12:03 PM
  • URL: https://libguides.bodleian.ox.ac.uk/Biology

Website feedback

Accessibility Statement - https://visit.bodleian.ox.ac.uk/accessibility

Google Analytics - Bodleian Libraries use Google Analytics cookies on this web site. Google Analytics anonymously tracks individual visitor behaviour on this web site so that we can see how LibGuides is being used. We only use this information for monitoring and improving our websites and content for the benefit of our users (you). You can opt out of Google Analytics cookies completely (from all websites) by visiting https://tools.google.com/dlpage/gaoptout

© Bodleian Libraries 2021. Licensed under a Creative Commons Attribution 4.0 International Licence

Stack Exchange Network

Stack Exchange network consists of 183 Q&A communities including Stack Overflow , the largest, most trusted online community for developers to learn, share their knowledge, and build their careers.

Q&A for work

Connect and share knowledge within a single location that is structured and easy to search.

Does the length of a PhD thesis matter?

I am writing a PhD thesis in literature. I have covered all areas in about 120 pages. Does the length of a PhD thesis matter even if it has covered all the areas mentioned in the synopsis?

  • research-process

user44894's user avatar

  • 4 Quality counts independant of length. –  Solar Mike Commented Jun 28, 2020 at 12:31
  • 4 Ask your PhD advisor; your question is country and university specific –  Basile Starynkevitch Commented Jun 28, 2020 at 12:32
  • 3 In some fields, it seems to do. Asking your PhD advisor is indeed your best bet. –  lighthouse keeper Commented Jun 28, 2020 at 12:33
  • Actually, if you want a weighty PhD thesis, then the grammage (thickness) of the paper that you print it on is just as important as the number of pages. If you printed those 120 pages on card-stock paper, it would be solid enough to knock the socks off any thesis supervisor. –  Tripartio Commented Oct 19, 2020 at 19:33

5 Answers 5

John Nash's thesis was 26 pages long with only two references and he later won a Nobel prize. What matters is scientific quality not quantity, if your ideas are superior nobody will object the length (mine was less than 100 pages).

Buzz's user avatar

  • 9 Math is really special in this respect. You won't find a PhD thesis in literature of that length. –  lighthouse keeper Commented Jun 28, 2020 at 14:50
  • 2 @lighthousekeeper But even that isn't because in literature theses are somehow evaluated by their length but only due to the fact that in literature its much harder to be concise when properly explaining a certain point. –  mlk Commented Jun 29, 2020 at 11:28
  • 1 @mlk That's a friendly, admissible interpretation of the situation. I'll leave it at that. –  lighthouse keeper Commented Jun 29, 2020 at 11:50
  • @lighthousekeeper true. You also won’t find someone who won the Nobel prize in literature for their PhD thesis, regardless of its length. –  Dan Romik Commented Jun 29, 2020 at 19:50

The advice you have so far is pretty US specific - in a system without a committee system, where you will be judged by your examiners in a one off thing, you may want to be more careful.

Of course, the person who should really know the answer to this is your supervisor/advisor, but should also be able to look up what the criteria are for your university. I would generally ask around your field. In my field in my country, a thesis is expected to contain at least three more or less complete "stories" that add up to a advance in the field. In biomolecular sciences, 120 pages would be very much on the short side for that, but not completely out of the question - a 40 page literature review and 20 pages of Methods, 5 pages of general discussion/conclusion would leave you with only 55 pages of results - doable if you are terse and everything worked first time.

But these expectations are going to differ from system to system and subject to subject. Speak to people in the know.

Ian Sudbery's user avatar

If it satisfies your advisor and the committee it is fine. A 1 page dissertation in poetry is within the realm of possibility, however unlikely.

It is a mistake to pad things, I think.

Some places (and some advisors) might have minimal length requirements, though I would question them. But if that is the issue here you need to think a bit more about the overall structure, not just the page count.

Buffy's user avatar

  • Could you perhaps expand on what you mean by “pad things” in your answer? The additional clarity I think could add value to the response. –  GrayLiterature Commented Jun 28, 2020 at 15:48
  • 1 @GrayLiterature, "padding" is a common idiom for adding less relevant things (or irrelevant) to make something bigger or longer. Think about a very thin person playing the typical Santa Claus role in a department store. (Or watch the NSFW movie "Bad Santa".) –  Buffy Commented Jun 28, 2020 at 15:52

I did one in the low hundred plus. As long as it passes your committee that is all that matters. But a brave face on, submit it and see the reaction.

Don't even ask if it is suitable. Just submit it to your advisor as your intended work product. After all your independent view is that it is good enough. Let him volunteer a criticism if he chooses. But don't suggest it.

guest's user avatar

  • 2 This only works in a system where you get the chance to try again if the committee doesn't like it. Not all systems are like that. –  Ian Sudbery Commented Jun 29, 2020 at 10:02

PhD theses are judged on the quality of their content. Length is only relevant to the extent that it makes that content understandable (not too short) and findable (not too long).

Anonymous Physicist's user avatar

You must log in to answer this question.

Not the answer you're looking for browse other questions tagged research-process thesis ..

  • Featured on Meta
  • Bringing clarity to status tag usage on meta sites
  • Announcing a change to the data-dump process

Hot Network Questions

  • Is it helpful to use a thicker gage wire for part of a long circuit run that could have higher loads?
  • What does "Two rolls" quote really mean?
  • How rich is the richest person in a society satisfying the Pareto principle?
  • When has the SR-71 been used for civilian purposes?
  • Current in a circuit is 50% lower than predicted by Kirchhoff's law
  • Confusion about time dilation
  • What does 'ex' mean in this context
  • Did Babylon 4 actually do anything in the first shadow war?
  • Replacing jockey wheels on Shimano Deore rear derailleur
  • Largest number possible with +, -, ÷
  • What is this movie aircraft?
  • Sum[] function not computing the sum
  • What should I consider when hiring a graphic designer to digitize my scientific plots?
  • What's the benefit or drawback of being Small?
  • Hashable and ordered enums to describe states of a process
  • How is carousing different from drunkenness in Luke 21:34-36? How should they be interpreted literally and spiritually?
  • What is the term for the belief that significant events cannot have trivial causes?
  • Gravitational potential energy of a water column
  • Can I use Cat 6A to create a USB B 3.0 Superspeed?
  • An instructor is being added to co-teach a course for questionable reasons, against the course author's wishes—what can be done?
  • Nausea during high altitude cycling climbs
  • "It never works" vs "It better work"
  • Can I Use A Server In International Waters To Provide Illegal Content Without Getting Arrested?
  • Euler should be credited with PNT?

biology phd thesis length

Recent Dissertation Titles

  • HDL in humans: a complex system of proteins, each with its own unique size distribution, metabolism, and diet regulation
  • MicroRNAs in Kidney Injury & Repair: Characterizing the Role of miR-132
  • Immunomodulatory pathways and metabolism
  • The evolution of drug resistant Mycobacterium tuberculosis
  • Empowering the experimental biology of Plasmodium vivax through elucidating requirements for ex vivo culture
  • Molecular mechanisms of metabolic adaptation to endurance exercise
  • Interplay of Trypanosoma cruzi and host metabolism and impact on parasite intracellular growth
  • A tale of brothers, sisters, aunts and uncles: using genomics and modeling to uncover the nature of P. falciparum polygenomic infections and cotransmission
  • Circadian integration of hepatic  de novo  lipogenesis and peripheral energy substrates utilization
  • Molecular and ecological factors modulating the fitness of Anopheles gambiae mosquitoes infected with Plasmodium falciparum
  • SREBP: A Key Effector of mTORC1 Signaling in Metabolism and Cancer
  • Where proteins go to die: Elucidating the physiological and therapeutic significance of the Clp protease complex in Mycobacterium tuberculosis
  • Mechanisms of recognition of HIV-infected cells by CD8+ T cells that mediate effective antiviral responses
  • Alterations in energy metabolism and neurodegeneration as a consequence of DNA damage
  • The Role of the Microbiome in Sex and Obesity Effects on Ozone-Induced Airway Hyperresponsiveness

News from the School

Red meat and diabetes

Red meat and diabetes

How for-profit medicine is harming health care

How for-profit medicine is harming health care

A tradition of mentoring

A tradition of mentoring

Promising HIV treatment

Promising HIV treatment

Digital Commons @ University of South Florida

  • USF Research
  • USF Libraries

Digital Commons @ USF > College of Arts and Sciences > Molecular Biosciences > Theses and Dissertations

Molecular Biosciences Theses and Dissertations

Theses/dissertations from 2024 2024.

Androgen Drives Melanoma Invasiveness and Metastatic Spread by Inducing Tumorigenic Fucosylation , Qian Liu

Theses/Dissertations from 2023 2023

Exploring strain variation and bacteriophage predation in the gut microbiome of Ciona robusta , Celine Grace F. Atkinson

Distinct Nrf2 Signaling Thresholds Mediate Lung Tumor Initiation and Progression , Janine M. DeBlasi

Thermodynamic frustration of TAD2 and PRR contribute to autoinhibition of p53 , Emily Gregory

Utilization of Detonation Nanodiamonds: Nanocarrier for Gene Therapy in Non-Small Cell Lung Cancer , Allan E. Gutierrez

Utilizing neoantigen-specific CD4* T cells and immune checkpoint modulation to advance adoptive cell therapy with tumor-infiltrating lymphocytes for metastatic melanoma patients , Maclean Scott Hall

Role of HLA-DRB1 Fucosylation in Anti-Melanoma Immunity , Daniel K. Lester

Targeting BET Proteins Downregulates miR-33a To Promote Synergy with PIM Inhibitors in CMML , Christopher T. Letson

The Role of the DNA Helicase Rrm3 under Replication Stress , Julius Muellner

Regulated Intramembrane Proteolysis by M82 Peptidases: The Role of PrsS in the Staphylococcus aureus Stress Response , Baylie M. Schott

Histone Deacetylase 8 is a Novel Therapeutic Target for Mantle Cell Lymphoma and Preserves Natural Killer Cell Cytotoxic Function , January M. Watters

Theses/Dissertations from 2022 2022

Ceramide-1-Phosphate: A Novel Regulator of Golgi Fragmentation, Golgi-ER Vesicle Trafficking, and Anaplasma phagocytophilum Pathogenesis , Anika Nayar Ali

Regulation of the Heat Shock Response via Lysine Acetyltransferase CBP-1 and in Neurodegenerative Disease in Caenorhabditis elegans , Lindsey N. Barrett

Establishment of a Melanoma ESC-GEMM Platform and Its Use to Study PTEN Tumor Suppressor Functions , Ilah Bok

Adrenergic Modulation of Precursor Cells of Ovarian Cancer , Sweta Dash

Determining the Role of Dendritic Cells During Response to Treatment with Paclitaxel/Anti-TIM-3 , Alycia Gardner

To be or not to be: A Tale of Staphylococcal GpsB , Lauren R. Hammond

Origin and Epigenetic Regulation of Cutaneous T Cell Lymphoma , Carly M. Harro

Cell-free DNA Methylation Signatures in Cancer Detection and Classification , Jinyong Huang

The Role Of Eicosanoid Metabolism in Mammalian Wound Healing and Inflammation , Kenneth D. Maus

A Holistic Investigation of Acidosis in Breast Cancer , Bryce Ordway

Characterizing the Impact of Postharvest Temperature Stress on Polyphenol Profiles of Red and White-Fruited Strawberry Cultivars , Alyssa N. Smith

Identification of Secondary Structural Elements Contained Within the Intrinsically Disordered N-Terminal Tail of the Bloom’s Syndrome Helicase. , Vivek Somasundaram

Defining the role of Oxidized Mitochondrial DNA in Myelodysplastic Syndromes , Grace Anne Ward

Lord of the Z-rings: Uncovering the Role of MraZ and FtsL in Bacillus subtilis Cell Division , Maria Louise White

Theses/Dissertations from 2021 2021

Multifaceted Approach to Understanding Acinetobacter baumannii Biofilm Formation and Drug Resistance , Jessie L. Allen

Cellular And Molecular Alterations Associated with Ovarian and Renal Cancer Pathophysiology , Ravneet Kaur Chhabra

Ecology and diversity of boletes of the southeastern United States , Arian Farid

CircREV1 Expression in Triple-Negative Breast Cancer , Meagan P. Horton

Microbial Dark Matter: Culturing the Uncultured in Search of Novel Chemotaxonomy , Sarah J. Kennedy

The Multifaceted Role of CCAR-1 in the Alternative Splicing and Germline Regulation in Caenorhabditis elegans , Doreen Ikhuva Lugano

Unraveling the Role of Novel G5 Peptidase Family Proteins in Virulence and Cell Envelope Biogenesis of Staphylococcus aureus , Stephanie M. Marroquin

Cytoplasmic Polyadenylation Element Binding Protein 2 Alternative Splicing Regulates HIF1α During Chronic Hypoxia , Emily M. Mayo

Transcriptomic and Functional Investigation of Bacterial Biofilm Formation , Brooke R. Nemec

A Functional Characterization of the Omega (ω) subunit of RNA Polymerase in Staphylococcus aureus , Shrushti B. Patil

The Role Of Cpeb2 Alternative Splicing In TNBC Metastasis , Shaun C. Stevens

Screening Next-generation Fluorine-19 Probe and Preparation of Yeast-derived G Proteins for GPCR Conformation and Dynamics Study , Wenjie Zhao

Theses/Dissertations from 2020 2020

Understanding the Role of Cereblon in Hematopoiesis Through Structural and Functional Analyses , Afua Adutwumwa Akuffo

To Mid-cell and Beyond: Characterizing the Roles of GpsB and YpsA in Cell Division Regulation in Gram-positive Bacteria , Robert S. Brzozowski

Spatiotemporal Changes of Microbial Community Assemblages and Functions in the Subsurface , Madison C. Davis

New Mechanisms That Regulate DNA Double-Strand Break-Induced Gene Silencing and Genome Integrity , Dante Francis DeAscanis

Regulation of the Heat Shock Response and HSF-1 Nuclear Stress Bodies in C. elegans , Andrew Deonarine

New Mechanisms that Control FACT Histone Chaperone and Transcription-mediated Genome Stability , Angelo Vincenzo de Vivo Diaz

Targeting the ESKAPE Pathogens by Botanical and Microbial Approaches , Emily Dilandro

Succession in native groundwater microbial communities in response to effluent wastewater , Chelsea M. Dinon

Role of ceramide-1 phosphate in regulation of sphingolipid and eicosanoid metabolism in lung epithelial cells , Brittany A. Dudley

Allosteric Control of Proteins: New Methods and Mechanisms , Nalvi Duro

Microbial Community Structures in Three Bahamian Blue Holes , Meghan J. Gordon

A Novel Intramolecular Interaction in P53 , Fan He

The Impact of Myeloid-Mediated Co-Stimulation and Immunosuppression on the Anti-Tumor Efficacy of Adoptive T cell Therapy , Pasquale Patrick Innamarato

Investigating Mechanisms of Immune Suppression Secondary to an Inflammatory Microenvironment , Wendy Michelle Kandell

Posttranslational Modification and Protein Disorder Regulate Protein-Protein Interactions and DNA Binding Specificity of p53 , Robin Levy

Mechanistic and Translational Studies on Skeletal Malignancies , Jeremy McGuire

Novel Long Non-Coding RNA CDLINC Promotes NSCLC Progression , Christina J. Moss

Genome Maintenance Roles of Polycomb Transcriptional Repressors BMI1 and RNF2 , Anthony Richard Sanchez IV

The Ecology and Conservation of an Urban Karst Subterranean Estuary , Robert J. Scharping

Biological and Proteomic Characterization of Cornus officinalis on Human 1.1B4 Pancreatic β Cells: Exploring Use for T1D Interventional Application , Arielle E. Tawfik

Evaluation of Aging and Genetic Mutation Variants on Tauopathy , Amber M. Tetlow

Theses/Dissertations from 2019 2019

Investigating the Proteinaceous Regulome of the Acinetobacter baumannii , Leila G. Casella

Functional Characterization of the Ovarian Tumor Domain Deubiquitinating Enzyme 6B , Jasmin M. D'Andrea

Integrated Molecular Characterization of Lung Adenocarcinoma with Implications for Immunotherapy , Nicholas T. Gimbrone

The Role of Secreted Proteases in Regulating Disease Progression in Staphylococcus aureus , Brittney D. Gimza

Advanced Proteomic and Epigenetic Characterization of Ethanol-Induced Microglial Activation , Jennifer Guergues Guergues

Understanding immunometabolic and suppressive factors that impact cancer development , Rebecca Swearingen Hesterberg

Biochemical and Proteomic Approaches to Determine the Impact Level of Each Step of the Supply Chain on Tomato Fruit Quality , Robert T. Madden

Enhancing Immunotherapeutic Interventions for Treatment of Chronic Lymphocytic Leukemia , Kamira K. Maharaj

Characterization of the Autophagic-Iron Axis in the Pathophysiology of Endometriosis and Epithelial Ovarian Cancers , Stephanie Rockfield

Understanding the Influence of the Cancer Microenvironment on Metabolism and Metastasis , Shonagh Russell

Modeling of Interaction of Ions with Ether- and Ester-linked Phospholipids , Matthew W. Saunders

Novel Insights into the Multifaceted Roles of BLM in the Maintenance of Genome Stability , Vivek M. Shastri

Conserved glycine residues control transient helicity and disorder in the cold regulated protein, Cor15a , Oluwakemi Sowemimo

A Novel Cytokine Response Modulatory Function of MEK Inhibitors Mediates Therapeutic Efficacy , Mengyu Xie

Novel Strategies on Characterizing Biologically Specific Protein-protein Interaction Networks , Bi Zhao

Theses/Dissertations from 2018 2018

Characterization of the Transcriptional Elongation Factor ELL3 in B cells and Its Role in B-cell Lymphoma Proliferation and Survival , Lou-Ella M.m. Alexander

Identification of Regulatory miRNAs Associated with Ethanol-Induced Microglial Activation Using Integrated Proteomic and Transcriptomic Approaches , Brandi Jo Cook

Molecular Phylogenetics of Floridian Boletes , Arian Farid

MYC Distant Enhancers Underlie Ovarian Cancer Susceptibility at the 8q24.21 Locus , Anxhela Gjyshi Gustafson

Quantitative Proteomics to Support Translational Cancer Research , Melissa Hoffman

A Systems Chemical Biology Approach for Dissecting Differential Molecular Mechanisms of Action of Clinical Kinase Inhibitors in Lung Cancer , Natalia Junqueira Sumi

Investigating the Roles of Fucosylation and Calcium Signaling in Melanoma Invasion , Tyler S. Keeley

Synthesis, Oxidation, and Distribution of Polyphenols in Strawberry Fruit During Cold Storage , Katrina E. Kelly

Investigation of Alcohol-Induced Changes in Hepatic Histone Modifications Using Mass Spectrometry Based Proteomics , Crystina Leah Kriss

Off-Target Based Drug Repurposing Using Systems Pharmacology , Brent M. Kuenzi

Investigation of Anemarrhena asphodeloides and its Constituent Timosaponin-AIII as Novel, Naturally Derived Adjunctive Therapeutics for the Treatment of Advanced Pancreatic Cancer , Catherine B. MarElia

The Role of Phosphohistidine Phosphatase 1 in Ethanol-induced Liver Injury , Daniel Richard Martin

Theses/Dissertations from 2017 2017

Changing the Pathobiological Paradigm in Myelodysplastic Syndromes: The NLRP3 Inflammasome Drives the MDS Phenotype , Ashley Basiorka

Modeling of Dynamic Allostery in Proteins Enabled by Machine Learning , Mohsen Botlani-Esfahani

Uncovering Transcriptional Activators and Targets of HSF-1 in Caenorhabditis elegans , Jessica Brunquell

The Role of Sgs1 and Exo1 in the Maintenance of Genome Stability. , Lillian Campos-Doerfler

Mechanisms of IKBKE Activation in Cancer , Sridevi Challa

Discovering Antibacterial and Anti-Resistance Agents Targeting Multi-Drug Resistant ESKAPE Pathogens , Renee Fleeman

Functional Roles of Matrix Metalloproteinases in Bone Metastatic Prostate Cancer , Jeremy S. Frieling

Disorder Levels of c-Myb Transactivation Domain Regulate its Binding Affinity to the KIX Domain of CREB Binding Protein , Anusha Poosapati

Role of Heat Shock Transcription Factor 1 in Ovarian Cancer Epithelial-Mesenchymal Transition and Drug Sensitivity , Chase David Powell

Cell Division Regulation in Staphylococcus aureus , Catherine M. Spanoudis

A Novel Approach to the Discovery of Natural Products From Actinobacteria , Rahmy Tawfik

Non-classical regulators in Staphylococcus aureus , Andy Weiss

Theses/Dissertations from 2016 2016

In Vitro and In Vivo Antioxidant Capacity of Synthetic and Natural Polyphenolic Compounds Identified from Strawberry and Fruit Juices , Marvin Abountiolas

Quantitative Proteomic Investigation of Disease Models of Type 2 Diabetes , Mark Gabriel Athanason

CMG Helicase Assembly and Activation: Regulation by c-Myc through Chromatin Decondensation and Novel Therapeutic Avenues for Cancer Treatment , Victoria Bryant

Advanced Search

  • Email Notifications and RSS
  • All Collections
  • USF Faculty Publications
  • Open Access Journals
  • Conferences and Events
  • Theses and Dissertations
  • Textbooks Collection

Useful Links

  • Rights Information
  • SelectedWorks
  • Submit Research

Home | About | Help | My Account | Accessibility Statement | Language and Diversity Statements

Privacy Copyright

Thesis and dissertation filing guidelines

Students who have enrolled in dissertation or thesis credits will prepare a manuscript to publish through ProQuest/UMI Dissertation Publishing. You own and retain the copyright to your manuscript. The Graduate School collects the manuscript via electronic submissions only. All manuscripts are made available through ProQuest Dissertations and Theses database (PQDT), in ProQuest/UMI’s Dissertation Abstracts International, and through the University’s institutional repository, ScholarWorks.

Getting started with campus resources:

  • Office of Human Research Protection
  • Campus computer Help Desk @One : (775) 682-5000
  • ProQuest Help Line: (877) 408-5027 (8 a.m. - 5 p.m. ET, or 5 a.m. - 2 p.m. PT)
  • For specific questions, call the Graduate School Graduation staff at (775) 784-6869

Jump to a section

  • Important dates and milestones for graduating students
  • Electronic manuscript submission
  • Checklist to complete your electronic submission
  • Instructions for completing thesis/dissertation committee approval page
  • Formatting your dissertation or thesis
  • Templates, samples and forms for filing

1. Important dates and milestones for graduating students

  • Contact your advisor to discuss department considerations and potential dates for your defense.
  • Contact the Graduate School to ensure your progression paperwork has been approved.
  • View important dates and purchase a graduation application through MyNevada for your graduation semester.
  • Doctoral students must submit their dissertation title for the commencement program.
  • Schedule defense date with the entire advisory committee in accordance with graduation deadlines.
  • Submit all forms and final manuscripts to the Graduate School by established deadlines.

2. Electronic Manuscript submission

ProQuest electronic submission site

Set up an account with ProQuest and wait for a password sent via email. ProQuest offers email and phone support,   1-877-408-5027 , frequently asked questions, etc. Visit the site early to familiarize yourself with the submission process.

3. Checklist to complete your electronic submission

  • Master's  Notice of Completion and Doctoral Notice of Completion Form  - This form includes all committee signatures AND the Graduate Program Director’s signature.
  • Master's Final Review Approval and Doctoral Final Review Approval   Form - This form serves as the final approval from your advisor. The Graduate School will accept the dissertation/thesis after the date listed on the form. The approval date on the form indicates the student’s submission can be accepted.
  • Committee Approval Page   - Use the online Word document template (NO SIGNATURES and no page number). This page will be merged into your manuscript to acknowledge committee members.
  • Filing for Copyright Registration   (optional) - Students have the opportunity to register a copyright of their graduate work with the U.S. Copyright Office. It is strictly optional, and there is a $75.00 fee associated with the service, which is paid online with student submission.
  • Processing fee  - $85 thesis / $95 dissertation.  Log into your Student Center in MyNEVADA . Under the Finances section, click on the link “Purchase Miscellaneous Items.” Select the applicable processing fee to pay (Dissertation or Thesis) and complete the transaction. You will receive a receipt that generates overnight.  Please keep this item as proof of payment for your records. Our office will automatically check for payment posted.
  • NSF Survey of Earned Doctorates  – For  doctoral students only.

4. Instructions for completing thesis/dissertation committee approval page

  • The Committee Approval Page (see forms links at bottom of page ): This interactive template has established borders.
  • Use the accompanying template on page two of this handout to complete the Committee Approval form. Check spelling carefully and make sure that case (upper-case/capital and lower-case letters) and font style (regular or bold) follow the template. Spacing between lines will depend on how long your thesis/dissertation title is and how many committee members you have.
  • Type the words as they appear on the template, i.e., on the first line “We recommend that the thesis/dissertation”, followed by the second line “prepared under our supervision by.”
  • At brackets [1] enter your full name in ALL CAPITAL LETTERS and BOLD-FACED.
  • Type the word “entitled” all in lowercase letters.
  • At brackets [2] enter the complete title of your thesis/dissertation. The title should be in both CAPITAL and lower-case letters and must be Bold-Faced. If the title is long, use two or more lines, breaking the lines at appropriate words in the title. Do not hyphenate between lines.
  • Type the words “be accepted in partial fulfillment of the,” and then, on the next line, “requirements for the degree of.”
  • At brackets [3] enter the name of the degree being awarded, e.g., for Ph.D. enter “Doctor of Philosophy,” for Ed.D. enter “Doctor of Education”. The degree should be in all CAPITAL LETTERS and Bold-Faced. DO NOT enter the name of the graduate program, such as anthropology or economics.
  • At brackets [4] type the full name of your thesis/dissertation advisor followed by his/her degree, followed by the word “Advisor”. For example, “Sonia A. Skakich, Ph.D., Advisor”. Use both capital and lowercase letters.
  • Enter the subsequent committee members and type the full names of the rest of your committee members followed by their degrees and their roles in the committee (Committee Member or Graduate School Rep.) under each one. Use one line for each member. The Graduate School Representative should be the last committee member listed. Use both capital and lowercase letters.
  • The last entry is reserved for the Dean of the Graduate School (which is already entered on the form).
  • At brackets [5] enter the month and year of official graduation. The month must be May, August, or December. Enter the appropriate four-digit designation of the year (e.g., 2018).

5. Formatting your dissertation or thesis

The Graduate School requires standardized formatting for the dissertation and thesis documents. Students will follow a style guide (APA, MLA, etc.) to prepare their document; however, the document must comply with University formatting requirements listed below.

Margins and spacing

  • Left margin: 1.5” from the left edge of the page.
  • Right margin: 1.0” from the right edge of the page.
  • Top margin: 1.0” from the top edge of the page.
  • Bottom margin: 1.25” from the bottom edge of the page.
  • All text should be double-spaced with the exception of captions, footnotes, long quotations, bibliographic entries of more than one line, and materials in tables and appendices.

Recommended fonts

Fonts should be easy to read. Times New Roman, Arial, or a similarly clear font is preferred; type size must be 10, 11, or 12 points. Script and italic typefaces are not acceptable except where absolutely necessary i.e. in Latin designations of species, etc.

In preparing your dissertation or thesis for electronic submission, you must embed all fonts. In Microsoft Word 2013, this is done by accessing the FILE menu; selecting OPTIONS, select SAVE. From the SAVE menu check the box labeled, ”Embed fonts in the file.” If the file size is a concern, check the box next to “Do NOT embed common system fonts."

Large tables, charts, etc., may be reduced to conform to page size, but the print must remain clear enough to be readable. You can also attach a PDF for electronic submissions.

Page numbering

Every page, with the exception of the title page, the copyright page, and the committee approval page is numbered in the upper right-hand corner, one-half inch from the top of the page and one inch from the right edge of the page. Do not underline or place a period after the number. Do not use a running header.

  • The prefatory materials (abstract, acknowledgments, table of contents, etc.) are numbered in lower case Roman numerals (i, ii, iii, iv…). Insert a section break after the Roman numerals to create different page numbering styles.
  • The first page of the main text and all subsequent pages are continuously numbered in Arabic numerals beginning with one until the final page number (1, 2, 3, 4…)
  • Do NOT number appendices or pages of additional material with numbers such as 4a or A-1.

Tables and appendices

Tables and appendices are part of the document and must conform to the same margin and page numbering requirements.

Format and sequence of pages

Assemble pages in the following order:

  • Title page *no page number* (create according to the example provided)
  • Copyright Notice *no page number* (optional - see example)
  • Committee Approval Page *no page number* (use the online template available on our   forms page – NO SIGNATURES on this page)
  • Abstract (begins lowercase Roman numerals i, ii, iii…)
  • Dedication (optional)
  • Acknowledgments (optional)
  • Table of Contents
  • List of Tables
  • List of Figures
  • Body of Manuscript (begins Arabic numbering 1, 2, 3…)
  • Back Matter (appendices, notes, bibliography, etc.)
  • Do not number the title page
  • Center each line of type
  • Use BOLD text type for the manuscript title
  • The date listed is the month and year in which you will graduate. The only acceptable months are May, August, and December (graduation cycles).

Copyright page

No page number on this page. Although not required, we strongly recommend you insert a copyright notice in your manuscript following the title page. Essential components of the copyright notice include the copyright symbol, full legal name of the author, and year of first publication. Follow the format of the sample provided below.

Committee approval page

  • No page number on this page
  • Use the electronic PDF template provided below. This page will list the advisory committee members and graduate dean but will NOT include committee signatures.   Combine the PDF into your manuscript to form a single PDF file.  To do this in Adobe Pro, select "Organize pages," "Insert," and "From file."   
  • A window will open and you can drag your separate PDF files into this window to combine them into a single file.
  • Choose the PDF documents in order of page sequencing (title page, committee page, main manuscript) and then combine files into a single PDF.

(Lower case Roman numeral “i” page number)

Abstracts are required for all theses and dissertations. ProQuest no longer has a word limit on the abstract, “as this constrains your ability to describe your research in a section that is accessible to search engines, and therefore would constrain potential exposure of your work.” ProQuest does publish print indices that include citations and abstracts of all dissertations and theses published by ProQuest/UMI. These print indices require word limits of 350 words for doctoral dissertations and 150 words for master’s theses (only text will be included in the abstract). You may wish to limit the length of your abstract if this concerns you. The abstracts as you submit them will NOT be altered in your published manuscript.

Processing note

Each copy of your thesis or dissertation will be checked for margins, clarity of copy, and pagination. The Graduate School will run the manuscript through the Turn It In plagiarism tool.

Electronically submitted theses/dissertations are available in electronic format only; no hard copies will be produced. Students are responsible for binding any copies for personal use or for distribution to their advisor, department, or committee members.

Dissertation & Thesis Processing Fee

Mandatory processing fees are required for all theses ($85.00) and all dissertations ($95.00). Log into your Student Center in MyNEVADA. Under the Finances section, click on the link “Purchase Miscellaneous Items.” Select the applicable processing fee to pay (Dissertation or Thesis) and complete the transaction. You will receive a receipt that generates overnight.  Please keep this item as proof of payment for your records. Our office will automatically check for payment posted.

Using copyrighted materials

You must certify in ProQuest that any copyrighted material used in your work, beyond brief excerpts, is with the written permission of the copyright owner. Attach copies of permission letters to the agreement form.

Copyright registration (optional)

Students have the opportunity to register a copyright on their graduate work with the U.S. Copyright Office. It is strictly optional, and there is a $75.00 fee associated with the service. Students submitting electronically pay online. Paying for the claim to copyright is a voluntary action, which allows a court of law to award monetary damages if the copyright is infringed. You may file a Registration of Copyright yourself by sending a properly completed application form, a nonrefundable filing fee of $45.00 and a nonreturnable copy of your thesis or dissertation to the United States Copyright Office. Application materials and instructions are available from:

Register of Copyrights Copyright Office Library of Congress Washington, D.C. 20559-6000 Information is also available at the Copyright Office’s website:   lcweb.loc.gov/copyright

ScholarWorks repository

ScholarWorks - the University's institutional repository - assists in collecting, preserving, and distributing the university's intellectual output accessible to end-users on local and global levels with few if any barriers. The repository will provide long-term access to the items deposited and can accept works from all the University faculty/staff/students. A wide variety of items including Articles, Datasets, Presentations, Technical Reports, Thesis and Dissertations, Posters, Conference Papers, etc. in all file formats can be deposited into the repository. The repository supports creative commons licensing and open-access publishing without any cost.

The discovery services and search engine optimizations ensure that major search engines easily discover the uploaded content. This increases the visibility, citations, and overall impact of the research. All items deposited in the repository receive a persistent URL that can be used for citations. Various statistics are collected with the built-in statistics module and Google Analytics modules. Information on monthly/yearly views, number of downloads, demographic information, etc. is available for each deposited item upon request.

All the ETDs uploaded into ProQuest are automatically deposited into the University's ScholarWorks repository. The embargo period set in ProQuest during deposit is carried over to the ScholarWorks repository. Any changes to the embargo period after deposit can be made by contacting ProQuest at 1-800-521-0600 as well as the ScholarWorks administrator at [email protected] .

Scholarworks FAQ

Do I need to upload my ETD into the ScholarWorks repository?

  • No, ProQuest will automatically upload the ETD into ScholarWorks on approval from the Graduate School.

Can I extend the embargo period on my Thesis/Dissertation after uploading it to ProQuest?

  • Yes, to change or extend the embargo period of your ETD you need to contact ProQuest at 1-800-521-0600 and the ScholarWorks administrator at [email protected] .

Can I make my ETD open access in the ScholarWorks repository?

  • Yes, ScholarWorks supports open access with creative commons licensing. It is available as a free service to all the faculty/staff/students.

Alternative formatting for thesis or dissertation

These guidelines apply to those theses or dissertations which consist of a number of papers either previously published or being published concurrently with the submission of the thesis or dissertation. Acceptance and publication of the articles are not criteria for this alternative. Each of the papers should constitute a separate chapter of the overall work. Preceding the papers should be an introductory section. This section may be one or more chapters but should include:

  • an overall introduction to the thesis/dissertation,
  • a review of the appropriate literature, and
  • a description of the methodology used in the study.

The student’s advisory committee should determine the format and specific content of this introductory section.

The number of individual papers constituting chapters of the thesis/dissertation is determined by the student’s advisory committee. These chapters may be formatted in the same style required by the journals to which they are to be submitted. However, the margins must conform to those of the overall thesis, i.e. left margin = 1.5"; right margin = 1"; top margin = 1"; bottom margin = 1.25". In addition, each page must be numbered consistent with the rest of the thesis/dissertation, that is, the first page of text is numbered 1 with each subsequent page numbered consecutively until the end, to include all appendices, indexes, etc.

Following the chapters consisting of individual papers, there must follow a summary, conclusions and recommendations section. This section may be formatted as one or more chapters.

Work reported in the articles should represent a major contribution by the student that is the review of the literature, the conceptual framework and/or research design for the reported work. The statistical analyses, summaries, conclusions, and recommendations should represent the student’s own work.

For publication purposes, other researchers may be named as additional authors. This would be especially appropriate when publication is dependent upon extensive revision of the initial manuscript submitted and the faculty involved assumes responsibility for the revisions, or when the student is using an existing database.

When a student chooses this option, the articles will be submitted to the journals agreed upon by the concerned academic unit. Responsibility for follow-up, revisions, etc., should be identified in a written document and agreed upon by the student and faculty member(s) involved.

6. Templates, samples and forms

Please be sure to read the above instructions before proceeding with documents.

Forms for filing a master's thesis   Forms for filing a doctoral dissertation

Thesis filing templates and samples

  • Committee Approval page for 3-member committee (TEMPLATE)
  • Committee Approval page for 3-member committee with co-advisor (TEMPLATE)
  • Committee Approval page for 4-member committee (TEMPLATE)
  • Committee Approval page for 4-member committee with co-advisor (TEMPLATE)

Sample pages

  • Thesis Title page (SAMPLE)
  • Thesis Copyright page (SAMPLE)
  • Thesis Committee approval page (SAMPLE)

Dissertation filing templates, samples and Survey of Earned Doctorates

  • Committee Approval page  for 5-member committee (TEMPLATE)
  • Committee Approval page  for  5-member committee with co-advisor (TEMPLATE)
  • Committee Approval page  for 6-member committee (TEMPLATE)
  • Committee Approval page  for 6 -member committee with co-advisor (TEMPLATE)
  • Dissertation Title page (SAMPLE)
  • Dissertation Copyright page (SAMPLE)
  • Dissertation Committee approval page (SAMPLE)

Survey of Earned Doctorates

  • Survey of Earned Doctorates  - The Survey of Earned Doctorates (SED) is an annual census conducted since 1957 of all individuals receiving a research doctorate from an accredited U.S. institution in a given academic year. The SED is sponsored by the National Center for Science and Engineering Statistics (NCSES) within the National Science Foundation (NSF) and by three other federal agencies: the National Institutes of Health, Department of Education, and National Endowment for the Humanities. The SED collects information on the doctoral recipient's educational history, demographic characteristics, and postgraduation plans. Results are used to assess characteristics of the doctoral population and trends in doctoral education and degrees. Read more about the purpose and methods of the SED .

X

UCL Division of Biosciences

Menu

overview information to go here about phds in CBD

Students are regularly monitored by a small committee appointed among experts in the field, which stimulates interactions between laboratories, and excellent courses are offered on generic research and other skills (speaking, writing, imaging, etc.).

max-mygfp-dac-hrp-lif-series

1. Programme Requirements  

2. funding options (link to new funding page), 3. how to apply, 4. self funded phd programmes, requirements for all cdb phd programmes.

To register for a PhD you will require one of the following:

- a first or upper second class UK Bachelor's degree in an appropriate subject, or an overseas qualification of equivalent standard from a recognised higher education institution - a recognised Master's degree If you are an overseas applicant you will also require evidence of proficiency in English . The standard of English expected is the 'good level'.

phd student and family at graduation

How to Apply

You are welcome to apply and commence studies throughout the year. Step 1: View the list of academic staff and study their research interests by visiting their web page and looking at some of their publications to decide whether you are interested in the research topics in that laboratory. Step 2: E-mail your chosen supervisor(s) to discuss possible projects. This will also ensure that they have space for you in their laboratory and that they will guide you in your application. Step 3: Complete an application form .  Select "Division of Biosciences" for the UCL Department you are applying to and "Research Degree: Cell and Developmental Biology" for Programme Title.

Self Funded PhD Programmes

Developmental and stem cell biology phd programme.

This programme offers a unique environment with one of the largest and strongest concentrations of high quality developmental and stem cell biologists anywhere in the world.

The programme provides training at the interface between developmental and stem cell biology. It is available either as a 3 or 4 year programme. The 4 year version includes a first year with a taught component and rotations in 3 laboratories providing the basis for an informed choice and an excellent platform for progression in the research project in years 2-4. Both include active, small group training in key skills including good writing, experimental design, reviewing papers and many modern techniques as appropriate to the project

First year: 3 rotations of 3 months each in different laboratories and journal clubs overseen by all supervisors on the programme (4 year programme only).

Second year: thesis work starts in August/September of the first year. In the second year, an advanced modern microscopy course and an intensive workshop to develop writing and reviewing skills take place in parallel, with a small group of students.

Third year: continue thesis research work, complemented with experimental design workshops.

Fourth year: complete research project, write up thesis. Workshops on funding, career planning and other skills tailored to each student.

The 3 year version of the programme starts from the "second year" above.

Please complete a UCL Graduate Student application form

Select "Division of Biosciences" for the UCL Department you are applying to, then for Programme Title:

  • 4 year programme "RRDDSCS4YR01 Research Degree: Developmental and Stem Cell Biology (4 years)"
  • 3 year programme "RRDDSCSING01 Research Degree: Developmental and Stem Cell Biology"

Please note re the application process:

  • For the 3 year programme, you can contact prospective supervisors and discuss a project before making your application.  For the 4 year programme, this isn't necessary.
  • When you submit your application, an e-mail request will automatically be sent to your referees providing them with a link to a secure website to upload a reference - please ensure that your referees add their references by the deadline. 
  • You will be required to upload your transcripts - if you are a current student and do not yet have a final transcript, please upload those from the years you have completed to date.  If you don't have a transcript, please upload a list of your marks in this space and forward the transcript to Admissions as soon as available.
  • If you are applying for a UCL Scholarship, please see the 'UCL Scholarships and Funded Programmes' box above as there are additional steps to complete.  For the 4 year programme, as this begins with lab rotations, the lay summary can be a brief outline of your research interests and there is no need to obtain a supervisor's reference as this will be added by the programme director if you are shortlisted. 

phylo image

Find a PhD Supervisor

If you already have funding in place or in preparation, the next step is to contact potential supervisors whose work interests you to design a project which you can propose on your UCL application form.

FEA Image

How much does a PhD cost?

Many of our students have independent funding, such as foreign government schemes, private support, small scholarships, and savings. Make sure you have enough to cover the 3 or 4 years a PhD takes.

CBER limits to adaptive change images

Accommodation

UCL provides around 6000 beds in 26 halls. Our halls vary in size, facilities and history.

origins of life image

Graduate Prospectus

Sections on networking, career destinations of our students and why study this degree

life sciences students in modern lab

Diversity and Inclusion

PhD students are part of a friendly, fair, and respectful GEE community. GEE embraces diversity and inclusion.

three students in biosciences lab

UCL Doctoral Skills Development Programme

The UCL Doctoral Skills Development Programme (UCL DocSkills) is designed to help you to expand your research and transferable skills in order to support your research, professional development and employability.

group of students in white lab coats

UKRI DTPs and CDTs

A majority of our PhD students are fully-funded for four years by Doctoral Training Programmes (DTPs) or Centres for Doctoral Training (CDTs) from UK Research and Innovation (UKRI) research councils, like NERC and BBSRC.

red bus in front of UCL Portico

Scholarships Finder

We have several students in receipt of UCL scholarships, including the prestigious Graduate Research Scholarship ( GRS ) and Overseas Research Scholarship ( ORS ), which provide full funding. Outstanding students are strongly encouraged to  contact potential supervisors  in research areas that interest them.

columns of the UCL Portico and statue

All UCL Funding Schemes

There are many possible sources of financial support for PhD students. This page gives a full description of funding for current and prospective PhD students at UCL.

grant museum

UK Doctoral Loan Scheme

A Postgraduate Doctoral Loan can help with course fees and living costs while you study a postgraduate doctoral course, such as a PhD.

2_phd_students_at_cber_at_a_desk

MPhil/PhD Handbook

This gives details of the support you will receive and the milestones you will pass during your PhD journey. CDB Thesis Committees comprise your primary supervisor, your secondary supervisor, and your tertiary supervisor, who usually acts as the committee chair. Each student upgrades from MPhil to PhD status after 12-18 months if approved by their Committee, and must submit their thesis after 3-4 years (5 years if part-time).

Te Here Tāura Rangahau

Faculty of --> faculty of graduate research, thesis length.

Research theses have a word limit that you must comply with.

A PhD thesis should not exceed a total of 100,000 words in length (or 70,000 for most professional doctorates), including scholarly apparatus such as footnotes or endnotes, essential appendices and bibliography. A doctoral thesis should however, be concise. Examiners often criticise excessive length, which frequently indicates poor judgement.

When you submit, you will be asked to certify that your thesis falls within the relevant word limit.

In exceptional circumstances, the Dean—Wellington Faculty of Graduate Research may grant permission for you to submit a longer thesis. You will need to apply for permission to exceed the word limit well in advance of submission.

Word limits

  • Doctor of Philosophy—100,000 words
  • Doctor of Government—70,000 words
  • Doctor of Health—70,000 words
  • Doctor of Nursing—70,000 words
  • Doctor of Midwifery—70,000 words
  • Doctor of Education—70,000 words
  • Doctor of Musical Arts—40,000 words
  • Skip to Main
  • CGSB Faculty
  • Affiliated Faculty
  • People in History
  • Genomics & Systems Biology
  • Developmental Genetics
  • Molecular & Cellular Biology
  • Computational Biology
  • Infectious Diseases and the Microbiome
  • Evolution, Ecology & Environmental Biology
  • Plant Genomics
  • Undergraduate Program
  • Master's Programs
  • PhD Program
  • Graduate Student Awards
  • Course Listings
  • Programs & Activities
  • Graduate Program Diversity
  • Mitra Lecture Series and Fellows
  • Conference for Undergraduate Research in Biology (CURB 2024)
  • Resource Guides
  • Rooms & Calendars

M.S. Qualifying Research Paper Writing Guidelines

An important part of your master’s education is the writing of your final master’s qualifying research paper (also called a master’s thesis). To aid you in this process we have assembled the following guidelines.

Qualifying research papers fall into two categories, those based on laboratory research and those based on literature research. Both types are mentored by a faculty member or principal investigator of a research laboratory sought by the student. For laboratory research, the student will write the paper based on original experimental results obtained in the laboratory of the mentor. For a literature-based thesis, after selection of a suitable topic by the student and mentor, the student will research the topic by reading and analyzing original literature on the subject, and then prepare a substantive analysis that will constitute the paper. All qualifying papers are graded "Pass", "Pass with Distinction" or "Fail". Further information can be found at the  M.S. in Biology website .

STYLE INFORMATION

Qualifying papers in the Department of Biology should follow the structure of the types of papers that appear in the journal Cell. Laboratory research based papers should be modeled after a Cell research style articles and library research based papers should be modeled after Cell review style articles. Example articles with links are listed in the notes below.  

    Laboratory Research
Based Paper
  Library Research
Based Paper
Style   Research article   Review Article
  You should include the following on your title page:
Your title should describe the main theme of your research project or topic.
  Limited to 200 words or less. Should summarize the main findings of your research.
  The introduction provides background information on the field of study and includes references to previously published works. You are describing where the research has come to this point.
  This section should be well developed/detailed and include all the laboratory techniques that you performed to obtain your results.   No methods section.
  This section comprises the experimental results you obtained in the lab.   No results section.
  No themes section.   This section contains a heading for each subtopic of your paper. where you describe in more detail the different specific findings. You may use figures in this section to illustrate your main points. Figures that come from published articles MUST be referenced in the figure legend.
  This section should be well developed/detailed and include your interpretation of your results and can include proposed models.   No discussion section.
  Optional if not fully covered in the discussion.   This section discusses the direction the field of study is headed and/or your ideas, hypotheses or proposals for future research.
  In the of the article you may reference text in parentheses in the format of (Smith et al., 2012) OR with a superscript number in the format of 23. See note on references.
In the list all references in the format of however they can be listed in alphabetical order OR in the order they appear in the text.
  Figures should be original and based on your research. It is best if figures are inserted into the appropriate place in the text rather than listed at the end of the paper.   If they come from a published article they MUST be referenced explicitly in the figure legend. It is best if figures are inserted into the appropriate place in the text rather than listed at the end of the paper.
  25-30 pages double spaced
  Your mentor may request a PDF or hard copy for grading. The Biology office requires a PDF version of your paper. Please do not submit hard copies to the Department. Your mentor will receive a grade sheet to use.

IMPORTANT NOTES

  • Plagiarism is an act or instance of using or closely imitating the language and thoughts of another author without authorization and the representation of that author's work as one's own, as by not crediting the original author: This means that you cannot copy lines of text from another source without noting that it is a quote (“put quotes around the text”) and citing the reference. In general avoid direct quotes from other sources unless the quote makes a strong point. General facts do not need to be referenced, but more specific facts and ideas must be referenced, even if they have been rewritten in your own words
  • Litertaure Review:  Lopez-Otin et al., 2023. Hallmarks of Aging: An Expanding Universe. Cell 186(2), 243-278 .
  • Research Research:  Lyons et al., 2023. Functional partitioning of transcriptional regulators by patterned charge blocks. Cell 186(2), 327-345 . 
  • Seek a thesis mentor and decide on a topic to study.
  • After beginning your literature research, provide a list of references you are reading to your mentor.
  • Proceed to write a brief outline of the paper with subsection ideas, then a more detailed outline with subsection descriptions, and share with mentor for comment.
  • Proceed to write a first draft of paper, which the mentor will read and provide suggestions/questions for comment.
  • Revise accordingly and submit the final version of your paper (~ 25 pages double spaced and reference list). Use citation format (Jones et al., 2023) and reference list as done in Cell papers (see above links). All figures taken from articles must be cited in the legend.

Secondary Menu

  • Thesis Guidelines

A thesis for Distinction in Biology is a wonderful way for you to close the loop on your undergraduate research experience and showcase your scientific scholarship. Your thesis will be evaluated by the Faculty in Biology and answers the following questions: What did you do? Why did you do it? What is the significance of your results? What else would you do, were you to continue the project?

In answering the above questions, you have an opportunity to demonstrate your understanding and intellectual ownership of a project; not simply your productivity in the lab. The volume of results or completeness of the study is not critical for a successful thesis. Instead, we will be looking for the following:

  • An argument for the significance of your research, contextualized within the scientific literature;
  • A review of appropriate literature as evidence in support of claims you make in your argument;
  • A statement of your research goals, i.e., a meaningful question of biological importance;
  • A description of experimental approaches and methods ;
  • Appropriate presentation of results through tables, figures, and images;
  • A discussion of the meaning and significance of your results;
  • A description of limitations and future directions for the project.

Expanded guidelines can be found in the Biology Thesis Assessment Protocol (BioTAP):

Format of the Thesis

The basic format of the thesis should resemble that of any scientific journal article that is common in your subdiscipline. It generally includes the following sections: Introduction & Background; Methods; Results; Discussion; Acknowledgements; and References. In some instances, it may be useful to sub-divide the Methods & Results section to correspond to multiple aims. However, if you chose to take this route, remember that there should still be a general Introduction and Discussion sections that address the project as a whole. The thesis should not consist of several "mini-papers" that are unconnected.  

Submission Guidelines

The format of the final copy should follow these guidelines:

  • Cover Page ( sample ): Title; student's name; supervisor's name; date of submission; 3 signature lines at bottom right (Research Supervisor, DUS, Reader). Please follow the format and language of the sample.
  • Abstract Page: single-spaced, roughly 250 words.
  • Thesis should be double-spaced
  • Pages should be numbered at the top right corner of the page
  • It is preferred that figures are embedded within the document instead of all at the end
  • There is no minimum page requirement or limit, although most are approximately 25 pages. 

Sample Theses

Examples of Distinction papers from previous years are available for examination in the Undergraduate Studies Office (Rm 135 BioSci).  Several samples are also available below as PDF files.

  • Tracing the origins of antimalarial resistance in Plasmodium vivax
  • Interaction network optimization improves the antimicrobial efficacy of phage cocktails
  • Identifying how ufmylation of RAB1B regulates IFN-β signaling

Additional Resources

  • Library Resources for Students Writing Theses
  • How to write and publish a scientific paper by Barbara Gastel and Robert A. Day
  • Biology 495(S): Scientific Argument in Writing . This course is particularly appropriate for seniors working on an undergraduate thesis or major research paper and is recommended, although not required, for all candidates for Graduation with Distinction in biology. The course is writing intensive and carries a “W” designation and, in the fall semester only, is a seminar and carries an “S” designation.
  • Biology Writes  offers writing resources, feedback, one-on-one consultations, and more.  
  • Duke Biology’s Mission Statement
  • AJED Annual and Semester Reports
  • AJED Meeting Notes
  • Biology Cultural Association (BCA)
  • Inclusion, Diversity, Equity, and Antiracism Committee (IDEA)
  • Learning from Baboons: Dr. Susan Alberts
  • Extremophiles and Systems Biology: Dr. Amy Schmid
  • How Cells Manage Stress: Dr. Gustavo Silva
  • Predator-Prey Interactions in a Changing World: Dr. Jean Philippe Gibert
  • Exploring the Extracellular Matrix: Dr. David Sherwood
  • Cell Division's Missing Link: Dr. Masayuki Onishi
  • Listening in to Birdsong: Dr. Steve Nowicki
  • Biogeochemistry as Ecosystem Accounting: Dr. Emily Bernhardt
  • Building a Dynamic Nervous System: Dr. Pelin Volkan
  • Investigating a Key Plant Hormone: Dr. Lucia Strader
  • Imagining Visual Ecology: Dr. Sönke Johnsen
  • Outreach Opportunities Across the Triangle
  • Job Opportunities
  • Location & Contact
  • Frequently Asked Questions
  • Learning Outcomes
  • Major Requirements
  • Anatomy, Physiology & Biomechanics
  • Animal Behavior
  • Biochemistry
  • Cell & Molecular Biology
  • Evolutionary Biology
  • Marine Biology
  • Neurobiology
  • Pharmacology
  • Plant Biology
  • Minor Requirements
  • Biology IDM
  • List of Biology Advisors
  • Guide for First-Year Students
  • Transfer Credit
  • Application & Deadlines
  • Supervisor & Faculty Reader
  • Honors Poster
  • Past Student Projects
  • Study Away Opportunities
  • Finding a Research Mentor
  • Project Guidelines
  • Getting Registered
  • Writing Intensive Study
  • Independent Study Abroad
  • Summer Opportunities
  • Departmental Awards
  • Biology Majors Union
  • Commencement 2024
  • Trinity Ambassadors
  • Degree Programs
  • Ph.D. Requirements
  • How to Apply
  • Financial Aid
  • Living in Durham
  • Where Our Students Go
  • Milestones Toward Ph.D.
  • Graduate School Fellowships
  • Useful Resources
  • Concurrent Biology Master of Science
  • En Route Biology Masters of Science
  • Form Library
  • Mentorship Expectations
  • On Campus Resources
  • Fellowships & Jobs
  • Meet Our Postdocs
  • Department Research Areas
  • Research Facilities
  • Duke Postdoctoral Association
  • All Courses
  • Biological Structure & Function Courses
  • Ecology Courses
  • Organismal Diversity Courses
  • Alternate Elective Courses
  • Primary Faculty
  • Secondary Faculty
  • Graduate Faculty
  • Emeritus Faculty
  • Graduate Students
  • Department Staff
  • Faculty Research Labs
  • Developmental Biology
  • Ecology & Population Biology
  • Neuroscience
  • Organismal Biology & Behavior
  • Systematics
  • Research Articles & Papers
  • Botany Plot
  • Field Station
  • Pest Management Protocols
  • Research Greenhouses
  • Centers/Research Groups
  • Biology Writes
  • Alumni Profiles
  • For Current Students
  • Assisting Duke Students

IMAGES

  1. How long is a PhD dissertation? [Data by field]

    biology phd thesis length

  2. Fillable Online Biology Thesis Guidelines and Template Fax Email Print

    biology phd thesis length

  3. Williams college biology thesis proposal sample

    biology phd thesis length

  4. Biology Honors Thesis Guidelines

    biology phd thesis length

  5. Guide to Write a PhD Thesis

    biology phd thesis length

  6. 119 Biology Thesis Topics For Your Masters Or Ph.D. Degree

    biology phd thesis length

VIDEO

  1. Top 15 Best Research Topics for microbiology for researchers and M.sc. students #study #yt #video

  2. Thesis Writing

  3. Key Mistakes to avoid during PhD Thesis Presentation

  4. Subjects for PhD Cancer Biology (Personal Experience)🧪 🧬 #studyabroad #phd #cancer

  5. Biology Thesis Presentation Animated Presentation Slides

  6. Convert a Thesis into a Research Article

COMMENTS

  1. PDF Guidelines for The PhD Dissertation

    Most dissertations are 100 to 300 pages in length. All dissertations should be divided into appropriate sections, and long dissertations may need chapters, main divisions, and even subdivisions. Students should keep in mind that GSAS and many departments deplore overlong and wordy dissertations.

  2. Average length of PhD dissertations by major : r/dataisbeautiful

    With a PhD in molecular and microbiology a good 70 pages of my dissertation is results, pictures of gels, etc. I love science. as a nursing student, I find nursing's position on the chart relative to say... neuroscience to be both reaffirming and intimidating. Yes!

  3. PDF PhD Program in Biological & Biomedical Sciences Guidelines and

    Overview of Training Timeline, Academic Components and Benchmarks for Degree Completion Year 1: Complete 5-6 semester-long courses along with course credit for completing rotations, complete 3 or more rotations, choose thesis lab. Complete the Year 1 IDP with a Curriculum Fellow/TF (affiliated with BBS 230A/B) and/or Program Advisor.

  4. How Long Is a PhD Thesis?

    Unfortunately, there's no one size fits all answer to this question. However, from the analysis of over 100 PhD theses, the average thesis length is between 80,000 and 100,000 words. A further analysis of 1000 PhD thesis shows the average number of pages to be 204. In reality, the actual word count for each PhD thesis will depend on the ...

  5. Doctor of Philosophy (PhD) in Biology

    The PhD in Biology is a research degree requiring graduate-level coursework, completion of a dissertation, and two semesters of participation in teaching (usually as a teaching fellow in laboratory or discussion sections of lecture courses led by Biology faculty). For most students, obtaining this degree typically involves five or more years of ...

  6. Why is a PhD thesis typically 150 pages? [closed]

    In summary, even if my thesis had nothing but 3 published/publishable manuscripts, it would already be >150 pages. I recognise that PhD theses vary in length; however in general theses have a substantially higher word count than a research publication. Number of papers as a marker of progress is annoying too.

  7. Word limits and requirements of your Degree Committee

    Biology. The thesis for the PhD is not to exceed 60,000 words in length (80,000 by special permission), exclusive of tables, footnotes, bibliography, and appendices. ... Applications for the limit of length of the thesis to be exceeded must be early — certainly no later than the time when the application for the appointment of examiners and ...

  8. How long is a PhD dissertation? [Data by field]

    A PhD can be anywhere from 50 pages to over 450 pages long. This equates to between about 20,000 words to 100,000 words. Most PhD theses are between 60,000 and 80,000 words long excluding contents, citations and references. A PhD thesis contains different sections including an introduction, methods, results and discussion, conclusions, further ...

  9. How long Should be PhD Dissertation? Stepwise Guide for 2024

    Balancing Depth and Brevity: Crafting a Dissertation Masterpiece. A. Emphasizing the Importance of Balance. B. Strategies for Maintaining Focus and Relevance. C. Encouraging Quality over Quantity. Tips for Managing PhD Dissertation Length: Navigating the Dissertation Writing Process. A. Effective Time Management.

  10. Tips for writing a PhD dissertation: FAQs answered

    A PhD thesis (or dissertation) is typically 60,000 to 120,000 words (100 to 300 pages in length) organised into chapters, divisions and subdivisions (with roughly 10,000 words per chapter) - from introduction (with clear aims and objectives) to conclusion. The structure of a dissertation will vary depending on discipline (humanities, social ...

  11. PhD and MPhil in Biology

    Your research. A PhD thesis can be up to 80,000 words long and must contain a substantial original contribution to scientific knowledge or understanding. An MPhil thesis can be up to 60,000 words and is expected to display a good general knowledge of the field of study; a comprehensive knowledge of some part or aspect of the field of study; and ...

  12. Information and Forms

    NSF Application - If eligible, Due October 18 (Life Sciences) or October 22 (Geosciences) TA Pre-Course Meeting Form - Due within the first two weeks of starting each TA position. TA Evaluation Form - Due after each TA position. Dissertation Lab Decision - Due May 1. First Year Evaluation and Planning Form - Due May 1. IDP Meeting - Due August 1.

  13. Rule of Three: advice on writing a PhD thesis

    Rule 2: prioritise papers and the thesis will follow. Papers are the priority. They are more useful to you and to your PI. But this advice isn't motivated by self-interest. If you go into the viva and the work in your thesis is already peer reviewed and published, it's harder for the examiners to criticise it.

  14. Thesis/dissertation committees

    Environmental Life Sciences PhD: End of 2nd semester; Evolutionary Biology PhD: End of 2nd semester; History and Philosophy of Science PhD: End of 4th semester and at least 1 semester prior to planned comprehensive exams and dissertation prospectus; Microbiology PhD: End of 3rd semester; Molecular and Cellular Biology PhD: End of 2nd semester ...

  15. Doctor of Philosophy

    Program requirements. A Doctor of Philosophy Degree requires three years full-time study and completion of a written thesis. The length of a doctoral thesis is normally around 100,000 words. The thesis is reviewed by members of the Australian and international scientific academic community. In the course of their research, PhD students must ...

  16. Biology: Theses and Dissertations

    Dissertation: In the UK, a dissertation is normally a document that presents an author's research findings as part of an undergraduate or master's programme. DPhil: An abbreviation for Doctor of Philosophy, which is an advanced research qualification. You may also see it referred to as PhD. ORA:

  17. Does the length of a PhD thesis matter?

    6. John Nash's thesis was 26 pages long with only two references and he later won a Nobel prize. What matters is scientific quality not quantity, if your ideas are superior nobody will object the length (mine was less than 100 pages). Math is really special in this respect.

  18. PhD Program in Biological Sciences in Public Health

    PhD Program in Biological Sciences in Public Health ... Recent Dissertation Titles. HDL in humans: a complex system of proteins, each with its own unique size distribution, metabolism, and diet regulation ... Empowering the experimental biology of Plasmodium vivax through elucidating requirements for ex vivo culture;

  19. Molecular Biosciences Theses and Dissertations

    Theses/Dissertations from 2016. PDF. In Vitro and In Vivo Antioxidant Capacity of Synthetic and Natural Polyphenolic Compounds Identified from Strawberry and Fruit Juices, Marvin Abountiolas. PDF. Quantitative Proteomic Investigation of Disease Models of Type 2 Diabetes, Mark Gabriel Athanason. PDF.

  20. Thesis and Doctoral Filing Guidelines

    Students who have enrolled in dissertation or thesis credits will prepare a manuscript to publish through ProQuest/UMI Dissertation Publishing. You own and retain the copyright to your manuscript. The Graduate School collects the manuscript via electronic submissions only. All manuscripts are made ...

  21. PhDs

    Third year: continue thesis research work, complemented with experimental design workshops. Fourth year: complete research project, write up thesis. Workshops on funding, career planning and other skills tailored to each student. The 3 year version of the programme starts from the "second year" above. Developmental and Stem Cell Biology PhD ...

  22. Thesis length

    A PhD thesis should not exceed a total of 100,000 words in length (or 70,000 for most professional doctorates), including scholarly apparatus such as footnotes or endnotes, essential appendices and bibliography. A doctoral thesis should however, be concise. Examiners often criticise excessive length, which frequently indicates poor judgement.

  23. M.S. Qualifying Research Paper Writing Guidelines

    An important part of your master's education is the writing of your final master's qualifying research paper (also called a master's thesis). To aid you in this process we have assembled the following guidelines. Qualifying research papers fall into two categories, those based on laboratory research and those based on literature research.

  24. Thesis Guidelines

    The format of the final copy should follow these guidelines: Cover Page (sample): Title; student's name; supervisor's name; date of submission; 3 signature lines at bottom right (Research Supervisor, DUS, Reader). Please follow the format and language of the sample. Abstract Page: single-spaced, roughly 250 words. Thesis should be double-spaced.